Решить неравенства с модулем онлайн с решением: Решение неравенств с модулем онлайн · Как пользоваться Контрольная Работа РУ

Содержание

Комплексные неравенства с модулем. Калькулятор онлайн.Решение уравнений и неравенств с модулями

решение неравенства в режиме онлайн решение почти любого заданного неравенства онлайн . Математические неравенства онлайн для решения математики. Быстро найти решение неравенства в режиме онлайн . Сайт www.сайт позволяет найти решение почти любого заданного алгебраического , тригонометрического или трансцендентного неравенства онлайн . При изучении практически любого раздела математики на разных этапах приходится решать неравенства онлайн . Чтобы получить ответ сразу, а главное точный ответ, необходим ресурс, позволяющий это сделать. Благодаря сайту www.сайт решение неравенства онлайн займет несколько минут. Основное преимущество www.сайт при решении математических неравенства онлайн — это скорость и точность выдаваемого ответа. Сайт способен решать любые алгебраические неравенства онлайн , тригонометрические неравенства онлайн , трансцендентные неравенства онлайн

, а также неравенства с неизвестными параметрами в режиме онлайн . Неравенства служат мощным математическим аппаратом решения практических задач. C помощью математических неравенств можно выразить факты и соотношения, которые могут показаться на первый взгляд запутанными и сложными. Неизвестные величины неравенств можно найти, сформулировав задачу на математическом языке в виде неравенств и решить полученную задачу в режиме онлайн на сайте www.сайт. Любое алгебраическое неравенство , тригонометрическое неравенство или неравенства содержащие трансцендентные функции Вы легко решите онлайн и получите точный ответ. Изучая естественные науки, неизбежно сталкиваешься с необходимостью решения неравенств . При этом ответ должен быть точным и получить его необходимо сразу в режиме
онлайн
. Поэтому для решения математических неравенств онлайн мы рекомендуем сайт www.сайт, который станет вашим незаменимым калькулятором для решения алгебраических неравенств онлайн , тригонометрических неравенств онлайн , а также трансцендентных неравенств онлайн или неравенств с неизвестными параметрами. Для практических задач по нахождению инетравол решений различных математических неравенств ресурса www.. Решая неравенства онлайн самостоятельно, полезно проверить полученный ответ, используя онлайн решение неравенств на сайте www.сайт. Необходимо правильно записать неравенство и моментально получите онлайн решение , после чего останется только сравнить ответ с Вашим решением неравенства. Проверка ответа займет не более минуты, достаточно решить неравенство онлайн и сравнить ответы. Это поможет Вам избежать ошибок в
решении
и вовремя скорректировать ответ при решении неравенств онлайн будь то алгебраическое , тригонометрическое , трансцендентное или неравенство с неизвестными параметрами.

Приложение

Решение неравенств онлайн на Math34.biz для закрепления студентами и школьниками пройденного материала. И тренировки своих практических навыков. Неравенство в математике — утверждение об относительной величине или порядке двух объектов (один из объектов меньше или не больше другого), или о том, что два объекта не одинаковы (отрицание равенства). В элементарной математике изучают числовые неравенства, в общей алгебре, анализе, геометрии рассматриваются неравенства также и между объектами нечисловой природы. Для решения неравенства обязательно должны быть определены обе его части с одним из знаков неравенства между ними. Строгие неравенства подразумевают неравенство двух объектов. В отличие от строгих, нестрогие неравенства допускают равенство входящих в него объектов. Линейные неравенства представляют собой простейшие с точки зрения начала изучения выражения, и для решения таких неравенств используются самые простые методики. Главная ошибка учеников в решении неравенств онлайн в том, что они не различают особенность строгого и нестрогого неравенства, от чего зависит войдут или нет граничные значения в конечный ответ. Несколько неравенств, связанных между собой несколькими неизвестными, называют системой неравенств. Решением неравенств из системы является некая область на плоскости, либо объемная фигура в трехмерном пространстве.

Наряду с этим абстрагируются n-мерными пространствами, однако при решении таких неравенств зачастую не обойтись без специальных вычислительных машин. Для каждого неравенства в отдельности нужно найти значения неизвестного на границах области решения. Множество всех решений неравенства и является его ответом. Замена одного неравенства равносильным ему другим неравенством называется равносильным переходом от одного неравенства к другому. Аналогичный подход встречается и в других дисциплинах, потому что помогает привести выражения к стандартному виду. Вы оцените по достоинству все преимущества решение неравенств онлайн на нашем сайте. Неравенство — это выражение, содержащее один из знаков = >. По сути это логическое выражение. Оно может быть либо верным, либо нет — в зависимости от того, что стоит справа и слева в этом неравенстве. Разъяснение смысла неравенства и основные приемы решения неравенств изучаются на разных курсах, а также в школе. Решение любых неравенств онлайн — неравенства с модулем, алгебраические, тригонометрические, трансцендентные неравенства онлайн.
Тождественное неравенство, как строгие и нестрогие неравенства, упрощают процесс достижения конечного результата, являются вспомогательным инструментом для разрешения поставленной задачи. Решение любых неравенств и систем неравенств, будь то логарифмические, показательные, тригонометрические или квадратных неравенства, обеспечивается с помощью изначально правильного подхода к этому важному процессу. Решение неравенств онлайн на сайте сайт всегда доступно всем пользователям и абсолютно бесплатно. Решениями неравенства с одной переменной называются значения переменной, которые обращают его в верное числовое выражение. Уравнения и неравенства с модулем: модуль действительного числа — это абсолютная величина этого числа. Стандартный метод решения этих неравенств заключается в возведении обеих частей неравенства в нужную степень. Неравенства – это выражения, указывающие на сравнение чисел, поэтому грамотное решение неравенств обеспечивает точность таких сравнений. Они бывают строгими (больше, меньше) и нестрогими (больше или равно, меньше или равно).
Решить неравенство – значит найти все те значения переменных, которые при подстановке в исходное выражение обращают его в верное числовое представление.. Понятие неравенства, его сущность и особенности, классификация и разновидности — вот что определяет специфику данного математического раздела. Основные свойства числовых неравенств, применимые ко всем объектам данного класса, обязательно должны быть изучены учениками на начальном этапе ознакомления с данной темой. Неравенства и промежутки числовой прямой очень тесно связаны, когда речь идет о решении неравенств онлайн. Графическое обозначение решения неравенства наглядно показывает суть такого выражения, становится понятно к чему следует стремиться при решении какой-либо поставленной задачи. В основу понятия неравенства входит сравнение двух или нескольких объектов. Неравенства, содержащие переменную, решаются как аналогично составленные уравнения, после чего делается выборка интервалов, которые будут приняты за ответ. Любое алгебраическое неравенство, тригонометрическое неравенство или неравенства содержащие трансцендентные функции, вы с легкостью и мгновенно сможете решить, используя наш бесплатный сервис.
Число является решением неравенства, если при подстановке этого числа вместо переменной получаем верное выражение, то есть знак неравенства показывает истинное понятие.. Решение неравенств онлайн на сайт каждый день для полноценного изучения студентами пройденного материала и закрепления своих практических навыков. Зачастую тема неравенства онлайн в математике изучается школьниками после прохождения раздела уравнений. Как и положено применяются все принципы при решении, чтобы определить интервалы решений. Найти в аналитическом виде ответ бывает сложнее, чем сделать то же самое, но в числовом виде. Однако такой подход дает более наглядное и полное представление об целостности решения неравенства. Сложность может возникнуть на этапе построения линии абсцисс и нанесения точек решения однотипного уравнения. После этого решение неравенств сводится к определению знака функции на каждом выявленном интервале с целью определения возрастания или убывания функции. Для этого необходимо поочередно подставлять к значениям, заключенных внутри каждого интервала, в исходную функцию и проверять её значение на положительность или отрицательность.
В этом есть суть нахождения всех решений, в том числе интервалов решений. Когда вы сами решите неравенство и увидите все интервалы с решениями, то поймете, насколько применим такой подход для дальнейших действий. Сайт сайт предлагает вам перепроверить свои результаты вычислений с помощью мощного современного калькулятора на этой странице. Вы сможете с легкостью выявить неточности и недочеты в своих расчетах, использую уникальный решебник неравенств. Студенты часто задаются вопросом, где найти такой полезный ресурс? Благодаря инновационному подходу к возможности определения потребностей инженеров, калькулятор создан на базе мощных вычислительных серверов с использованием только новых технологий. По сути решение неравенств онлайн заключается в решении уравнения с вычислением всех возможных корней. Полученные решения отмечаются на прямой, а далее производится стандартная операция по определению значения функции на каждом промежутке. А что же делать, если корни уравнения получаются комплексные, как в этом случае решить неравенство в полной форме, которое бы удовлетворяло всем правилам написания результата? Ответ на этот и многие другие вопросы с легкость даст наш сервис сайт, для которого нет ничего невозможного в решении математических задач онлайн.
В пользу вышесказанного добавим следующее: каждый, кто всерьез занимается изучением такой дисциплиной как математика, обязан изучить тему неравенств. Неравенства бывают разных типов и решить неравенство онлайн порой сделать непросто, так как необходимо знать принципы подходов к каждому из них. На этом базируется основа успеха и стабильности. Для примера можно рассмотреть такие типы, как логарифмические неравенства или трансцендентные неравенства. Это вообще особый вид таких, сложных на первый взгляд, задач для студентов, тем более для школьников. Преподаватели институтов уделяют немало времени из подготовки практикантов для достижения профессиональных навыков в работе. К таким же типам отнесем тригонометрические неравенства и обозначим общий подход при решении множества практических примеров из постановочной задачи. В ряде случаев сначала нужно привести все к уравнению, упростить его, разложить на разные множители, короче говоря, привести к вполне наглядному виду. Во все времена человечество стремилось найти оптимальный подход в любых начинаниях. Благодаря современным технологиям, человечество сделало просто огромный прорыв в будущее свое развитие. Инновации все чаще и чаще, день за днем вливаются в нашу жизнь. В основу вычислительной техники легла, разумеется, математика со своим принципами и строгим подходом к делу. сайт представляет собой общий математический ресурс, в котором имеется разработанный калькулятор неравенств и многие другие полезные сервисы. Используйте наш сайт и у вас будет уверенность в правильности решенных задач. Из теории известно, что объекты нечисловой природы также изучаются неравенствами онлайн, только этот подход представляет собой особый способ изучения данного раздела в алгебре, геометрии и других направлениях математики. Решать неравенства можно по-разному, неизменным остается конечная проверка решений и лучше всего это делать прямой подстановкой значений в само неравенство. Во многих случаях полученный ответ очевиден и его легко проверить в уме. Предположим нам задано решить дробное неравенство, в котором присутствуют искомые переменные в знаменателях дробных выражений. Тогда решение неравенств сведется к приведению всех слагаемых к общему знаменателю, предварительно переместив все в левую и правую часть неравенства. Далее нужно решить однородное уравнение, полученное в знаменателе дроби. Эти числовые корни будут точками, не включенными в интервалы общего решения неравенства, или ка их еще называют — проколотые точки, в которых функция обращается в бесконечность, то есть функция не определена, а можно только получить ее предельное значение в данной точке. Решив полученное в числителе уравнение, все точки нанесем на числовую ось. Заштрихуем те точки, в которых числитель дроби обращаемся в ноль. Соответственно все остальные точки оставляем пустыми или проколотыми. Найдем знак дроби на каждом интервале и после этого выпишем окончательный ответ. Если на границах интервала будут заштрихованные точки, то тогда включаем эти значения в решение. Если на границах интервала будут проколотые точки — эти значения в решение не включаем. После того, как решите неравенство, вам потребуется в обязательном порядке проверить полученный результат. Можно это сделать руками, каждое значение из интервалов ответа поочередно подставить в начальное выражение и выявить ошибки. Сайт сайт с легкостью выдаст вам все решения неравенства, и вы сразу сравните полученные вами и калькулятором ответы. Если все-таки ошибка будет иметь место, то на нашем ресурсе решение неравенств онлайн окажется вам очень полезным. Рекомендуем всем студентам вначале приступать не к решению напрямую неравенства, а сначала получить результат на сайт, потому что в дальнейшем будет намного проще самому сделать правильный расчет. В текстовых задачах практически всегда решение сводится к составлению системы неравенств с несколькими неизвестными. Решить неравенство онлайн в считанные секунды поможет наш ресурс. При этом решение будет произведено мощной вычислительной программой с высокой точностью и без всяких погрешностей в конечном ответе. Тем самым вы сможете сэкономить колоссальное количество времени на решении данным калькулятором примеров. В ряде случаев школьники испытывают затруднения, когда на практике или в лабораторных работах встречают логарифмические неравенства, а еще хуже, когда видят перед собой тригонометрические неравенства со сложными дробными выражениями с синусами, косинусами или вообще с обратными тригонометрическими функциями. Как ни крути, но без помощи калькулятора неравенств справиться будет очень сложно и не исключены ошибки на любом этапе решения задачи. Пользуйтесь ресурсом сайт совершенно бесплатно, он доступен каждому пользователю каждый день. Начинать действовать с нашего сервиса-помощника очень хорошая идея, поскольку аналогов существует множество, а по-настоящему качественных сервисов единицы. Мы гарантируем точность вычислений при длительности поиска ответа в несколько секунд. От вас требуется только записать неравенства онлайн, а мы в свою очередь сразу предоставим вам точный результат решения неравенства. Искать подобный ресурс может оказаться бессмысленным занятием, так как вряд ли вы встретите такой же качественный сервис как у нас. Можно обойтись без теории про решение неравенств онлайн, но без качественного и быстрого калькулятора вам не обойтись. Желаем вам успехов в учебе! По-настоящему выбрать оптимальное решение неравенства онлайн зачастую связано с логическим подходом для случайной величины. Если пренебречь малым отклонением замкнутого поля, то вектор нарастающего значения пропорционален наименьшему значению на промежутке убывания линии ординат. Инвариант пропорционален двукратному увеличению отображаемым функциям наряду с исходящим ненулевым вектором. Лучший ответ всегда содержит точность вычислений. Наше решение неравенств примет вид однородной функции последовательно сопряженных числовых подмножеств главного направления. За первый интервал возьмем как раз наихудшее по точности значение нашего представления переменной. Вычислим на максимальное отклонение предыдущее выражение. Будем пользоваться сервисом на усмотрение предложенных вариантов по мере необходимости. Будет ли найдено решение неравенств онлайн с помощью хорошего в своем классе калькулятора — это риторический вопрос, разумеется, студентам такой инструмент пойдет только на пользу и принесет огромный успех в математике. Наложим ограничение на область с множеством, которое сведем к элементам с восприятием импульсов по напряжению. Физические значения таких экстремумов математически описывают возрастание и убывание кусочно-непрерывных функций. На протяжении всего пути ученые находили доказательства существования элементов на разных уровнях изучения. Расположим все последовательно идущие подмножества одного комплексного пространства в один ряд с такими объектами, как шар, куб или цилиндр. Из нашего результата можно сделать однозначный вывод и когда решите неравенство, то на выходе, безусловно, прольется свет на высказанное математическое предположение об интеграции метода на практике. В текущем положении вещей необходимое условие будет также являться и достаточным условием. Критерии неопределенности зачастую вызывают у студентов разногласия по причине недостоверных данных. Это упущение должны взять на себя преподаватели ВУЗов, а также учителя в школах, так как на начальном этапе обучения необходимо это тоже учитывать. Из вышесказанного вывода на взгляд опытных людей можно делать выводы, что решить неравенство онлайн очень сложное задание при вхождении в неравенство неизвестных разного типа данных. Об этом сказано на научной конференции в западном округе, на которой выдвигали самые различные обоснования по поводу научных открытий в области математики и физики, а также молекулярного анализа биологически устроенных систем. В нахождении оптимального решения абсолютно все логарифмические неравенства представляют научную ценность для всего человечества. Исследуем данный подход на предмет логических заключений по ряду несовпадений на высшем уровне понятий о существующем объекте. Логика подсказывает иное, чем видно на первый взгляд неопытному студенту. По причине возникновения масштабных аналогий, будет рационально сначала приравнять отношения к разности предметов исследуемой области, а затем показать на практике наличие общего аналитического результата. Решение неравенств абсолютным образом завязано на применении теории и будет важно для каждого изучить такой необходимый для дальнейших исследований раздел математики. Однако, при решении неравенств вам нужно найти все корни составленного уравнения, а уже затем нанести все точки на ось ординат. Некоторые точки будут проколоты, а остальные войдут в интервалы с общим решением. Начнем изучать раздел математики с азов важнейшей дисциплины школьной программы. Если тригонометрические неравенства являются неотъемлемой частью текстовой задачи, то, как раз применять ресурс для вычисления ответа просто необходимо. Введите левую и правую части неравенства корректно, нажмите на кнопу и получите результат в течение нескольких секунд. Для быстрых и точных математических вычислений с числовыми или символьными коэффициентами перед неизвестными, вам как всегда понадобится универсальный калькулятор неравенств и уравнений, который сможет в считанные секунды предоставить ответ на поставленную вами задачку. Если у вас нет времени на написание целого ряда письменных упражнений, то обоснованность сервиса неоспорима даже невооруженным глазом. Для студентов такой подход является более оптимальным и оправданным с точки зрения экономии материальных ресурсов и времени. Напротив катета лежит угол, а для его измерения необходим циркуль, но вы сможете в любо момент воспользоваться подсказками и решите неравенство не применяя никаких формул приведения. Означает ли это успешное завершение начатого действия? Однозначно ответ будет положительным.

Модулем числа называется само это число, если оно неотрицательное, или это же число с противоположным знаком, если оно отрицательное.

Например, модулем числа 6 является 6, модулем числа -6 тоже является 6.

То есть под модулем числа понимается абсолютная величина, абсолютное значение этого числа без учета его знака.

Обозначается так: |6|, |х |, |а | и т.д.

(Подробнее — в разделе «Модуль числа»).

Уравнения с модулем.

Пример 1 . Решить уравнение |10 х — 5| = 15.

Решение .

В соответствии с правилом, уравнение равносильно совокупности двух уравнений:

10х — 5 = 15
10х — 5 = -15

Решаем:

10х = 15 + 5 = 20
10х = -15 + 5 = -10

х = 20: 10
х = -10: 10

х = 2
х = -1

Ответ : х 1 = 2, х 2 = -1.

Пример 2 . Решить уравнение |2 х + 1| = х + 2.

Решение .

Поскольку модуль — число неотрицательное, то х + 2 ≥ 0. Соответственно:

х ≥ -2.

Составляем два уравнения:

2х + 1 = х + 2
2х + 1 = -(х + 2)

Решаем:

2х + 1 = х + 2
2х + 1 = —х — 2

2х х = 2 — 1
2х + х = -2 — 1

х = 1
х = -1

Оба числа больше -2. Значит, оба являются корнями уравнения.

Ответ : х 1 = -1, х 2 = 1.

Пример 3 . Решить уравнение

|х + 3| — 1
————— = 4
х — 1

Решение .

Уравнение имеет смысл, если знаменатель не равен нулю — значит, если х ≠ 1. Учтем это условие. Наше первое действие простое — не просто освобождаемся от дроби, а преобрахуем ее так, чтобы получить модуль в чистом виде:

|х + 3| — 1 = 4 · (х — 1),

|х + 3| — 1 = 4х — 4,

|х + 3| = 4х — 4 + 1,

|х + 3| = 4х — 3.

Теперь у нас в левой части уравнения только выражение под модулем. Идем дальше.
Модуль числа есть неотрицательное число — то есть он должен быть больше нуля или равен нулю. Соответственно, решаем неравенство:

4х — 3 ≥ 0

4х ≥ 3

х ≥ 3/4

Таким образом, у нас появилось второе условие: корень уравнения должен быть не меньше 3/4.

В соответствии с правилом, составляем совокупность двух уравнений и решаем их:

х + 3 = 4х — 3
х + 3 = -(4х — 3)

х + 3 = 4х — 3
х + 3 = -4х + 3

х — 4х = -3 — 3
х + 4х = 3 — 3

х = 2
х = 0

Мы получили два ответа. Проверим, являются ли они корнями исходного уравнения.

У нас было два условия: корень уравнения не может быть равен 1, и он должен быть не меньше 3/4. То есть х ≠ 1, х ≥ 3/4. Обоим этим условиям соответствует только один из двух полученных ответов — число 2. Значит, только оно и является корнем исходного уравнения.

Ответ : х = 2.

Неравенства с модулем.

Пример 1 . Решить неравенство | х — 3|

Решение .

Правило модуля гласит:

|а | = а , если а ≥ 0.

|а | = —а , если а

Модуль может иметь и неотрицательное, и отрицательное число. Значит, мы должны рассмотреть оба случая: х — 3 ≥ 0 и х — 3

1) При х — 3 ≥ 0 наше исходное неравенство остается как есть, только без знака модуля:
х — 3

2) При х — 3

-(х — 3)

Раскрыв скобки, получаем:

х + 3

Таким образом, от этих двух условий мы пришли к объединению двух систем неравенств:

х — 3 ≥ 0
х — 3

х — 3 —х + 3

Решим их:

х ≥ 3
х

х х > -1

Итак, у нас в ответе объединение двух множеств:

3 ≤ х х

Определяем наименьшее и наибольшее значения. Это -1 и 7. При этом х больше -1, но меньше 7.
Кроме того, х ≥ 3. Значит, решением неравенства является все множество чисел от -1 до 7, исключая эти крайние числа.

Ответ : -1 х

Или: х ∈ (-1; 7).

Дополнения .

1) Есть более простой и короткий способ решения нашего неравенства — графический. Для этого надо нарисовать горизонтальную ось (рис.1).

Выражение |х — 3| х до точки 3 меньше четырех единиц. Отмечаем на оси число 3 и отсчитываем влево и вправо от от него 4 деления. Слева мы придем к точке -1, справа — к точке 7. Таким образом, точки х мы просто увидели, не вычисляя их.

При этом, согласно условию неравенства, сами -1 и 7 не включены во множество решений. Таким образом, получаем ответ:

1 х

2) Но есть еще одно решение, которое проще даже графического способа. Для этого наше неравенство надо представить в следующем виде:

4 х — 3

Ведь так оно и есть по правилу модуля. Неотрицательное число 4 и аналогичное отрицательное число -4 являются границами решения неравенства.

4 + 3 х

1 х

Пример 2 . Решить неравенство | х — 2| ≥ 5

Решение .

Этот пример существенно отличается от предыдущего. Левая часть больше 5 либо равна 5. С геометрической точки зрения, решением неравенства являются все числа, которые от точки 2 отстоят на расстоянии 5 единиц и больше (рис.2). По графику видно, что это все числа, которые меньше или равны -3 и больше или равны 7. А значит, мы уже получили ответ.

Ответ : -3 ≥ х ≥ 7.

Попутно решим это же неравенство способом перестановки свободного члена влево и вправо с противоположным знаком:

5 ≥ х — 2 ≥ 5

5 + 2 ≥ х ≥ 5 + 2

Ответ тот же: -3 ≥ х ≥ 7.

Или: х ∈ [-3; 7]

Пример решен.

Пример 3 . Решить неравенство 6 х 2 — | х | — 2 ≤ 0

Решение .

Число х может быть и положительным числом, и отрицательным, и нулем. Поэтому нам надо учесть все три обстоятельства. Как вы знаете, они учитываются в двух неравенствах: х ≥ 0 и х х ≥ 0 мы просто переписываем наше исходное неравенство как есть, только без знака модуля:

6х 2 — х — 2 ≤ 0.

Теперь о втором случае: если х отрицательного числа является это же число с противоположным знаком. То есть пишем число под модулем с обратным знаком и опять же освобождаемся от знака модуля:

6х 2 — (-х ) — 2 ≤ 0.

Раскрываем скобки:

6х 2 + х — 2 ≤ 0.

Таким образом, мы получили две системы уравнений:

6х 2 — х — 2 ≤ 0
х ≥ 0

6х 2 + х — 2 ≤ 0
х

Надо решить неравенства в системах — а это значит, надо найти корни двух квадратных уравнений. Для этого приравняем левые части неравенств к нулю.

Начнем с первого:

6х 2 — х — 2 = 0.

Как решается квадратное уравнение — см. раздел «Квадратное уравнение». Мы же сразу назовем ответ:

х 1 = -1/2, х 2 = 2/3.

Из первой системы неравенств мы получаем, что решением исходного неравенства является все множество чисел от -1/2 до 2/3. Пишем объединение решений при х ≥ 0:
[-1/2; 2/3].

Теперь решим второе квадратное уравнение:

6х 2 + х — 2 = 0.

Его корни:

х 1 = -2/3, х 2 = 1/2.

Вывод: при х

Объединим два ответа и получим итоговый ответ: решением является все множество чисел от -2/3 до 2/3, включая и эти крайние числа.

Ответ : -2/3 ≤ х ≤ 2/3.

Или: х ∈ [-2/3; 2/3].

Существует несколько способов решения неравенств, содержащих модуль. Рассмотрим некоторые из них.

1) Решение неравенства с помощью геометрического свойства модуля.

Напомню, что такое геометрическое свойство модуля: модуль числа x – это расстояние от начала координат до точки с координатой x.

В ходе решения неравенств этим способом может возникнуть 2 случая:

1. |x| ≤ b,

И неравенство с модулем очевидно сводится к системе двух неравенств. Тут знак может быть и строгим, в этом случае точки на картинке будут «выколотыми».

2. |x| ≥ b, тогда картинка решения выглядит так:

И неравенство с модулем очевидно сводится к совокупности двух неравенств. Тут знак может быть и строгим, в этом случае точки на картинке будут «выколотыми».

Пример 1.

Решить неравенство |4 – |x|| 3.

Решение.

Данное неравенство равносильно следующей совокупности:

U [-1;1] U

Пример 2.

Решить неравенство ||x+2| – 3| 2.

Решение.

Данное неравенство равносильно следующей системе.

{|x + 2| – 3 ≥ -2
{|x + 2| – 3 ≤ 2,
{|x + 2| ≥ 1
{|x + 2| ≤ 5.

Решим отдельно первое неравенство системы. Оно эквивалентно следующей совокупности:

U [-1; 3].

2) Решение неравенств, используя определение модуля.

Напомню для начала определение модуля.

|a| = a, если a 0 и |a| = -a, если a

Например, |34| = 34, |-21| = -(-21) = 21.

Пример 1.

Решить неравенство 3|x – 1| x + 3.

Решение.

Используя определение модуля получим две системы:

{x – 1 ≥ 0
{3(x – 1) ≤ x + 3

{x – 1 {-3(x – 1) ≤ x + 3.

Решая первую вторую системы в отдельности, получим:

{x ≥ 1
{x ≤ 3,

{x {x ≥ 0.

Решением исходного неравенства будут все решения первой системы и все решения второй системы.

Ответ: x € .

3) Решение неравенств методом возведения в квадрат.

Пример 1.

Решить неравенство |x 2 – 1|

Решение.

Возведем обе части неравенства в квадрат. Замечу, что возводить обе части неравенства в квадрат можно только в том случае, когда они обе положительные. В данном случае у нас и слева и справа стоят модули, поэтому мы можем это сделать.

(|x 2 – 1|) 2

Теперь воспользуемся следующим свойством модуля: (|x|) 2 = x 2 .

(x 2 – 1) 2

(x 2 – 1) 2 – (x 2 – x + 1) 2

(x 2 – 1 – x 2 + x – 1)(x 2 – 1 + x 2 – x + 1)

(x – 2)(2x 2 – x)

x(x – 2)(2x – 1)

Решаем методом интервалов.

Ответ: x € (-∞; 0) U (1/2; 2)

4) Решение неравенств методом замены переменных.

Пример.

Решить неравенство (2x + 3) 2 – |2x + 3| 30.

Решение.

Заметим, что (2x + 3) 2 = (|2x + 3|) 2 . Тогда получим неравенство

(|2x + 3|) 2 – |2x + 3| ≤ 30.

Сделаем замену y = |2x + 3|.

Перепишем наше неравенство с учетом замены.

y 2 – y ≤ 30,

y 2 – y – 30 ≤ 0.

Разложим квадратный трехчлен, стоящий слева, на множители.

y1 = (1 + 11) / 2,

y2 = (1 – 11) / 2,

(y – 6)(y + 5) ≤ 0.

Решим методом интервалов и получим:

Вернемся к замене:

5 ≤ |2x + 3| ≤ 6.

Данное двойное неравенство равносильно системе неравенств:

{|2x + 3| ≤ 6
{|2x + 3| ≥ -5.

Решим каждое из неравенств в отдельности.

Первое равносильно системе

{2x + 3 ≤ 6
{2x + 3 ≥ -6.

Решим ее.

{x ≤ 1.5
{x ≥ -4.5.

Второе неравенство очевидно выполняется для всех x, так как модуль по определению число положительное. Так как решение системы – это все x, которые удовлетворяют одновременно и первому и второму неравенству системы, то решением исходной системы будет решение ее первого двойного неравенства (ведь второе верно для всех x).

Ответ: x € [-4,5; 1,5].

blog.сайт, при полном или частичном копировании материала ссылка на первоисточник обязательна.

Неравенства с модулем и их решение

Определение и формулы неравенств с модулем

ОПРЕДЕЛЕНИЕ

Модуль числа a равен числу a, если число положительное и -a, если оно отрицательное.

Можно записать следующим образом, что

   

Модуль числа — это расстояние от нуля до данного числа.

Если под модулем находится функция , то

1. Неравенство вида равносильно системе неравенств , при условии ; при — решений нет.

2. Неравенство вида равносильно совокупности неравенств , при условии, что . Если , то неравенство справедливо при всех допустимых значениях .

3. Неравенство равносильно двойному неравенству .

4. Неравенство равносильно совокупности неравенств

5. Неравенство вида выполняется тогда и только тогда, когда

   

Примеры решения неравенств с модулем

ПРИМЕР 2
Задание Решить неравенство
Решение Нулями подмодульных выражений являются значения и , которые разбивают числовую ось на три интервала.
  1. Если , то заданное неравенство принимает вид:

       

    В этом случае решений нет, так как получили неверное неравенство, то есть .

  2. Если , то

    Пересечением интервала, на котором рассматривается заданное неравенство, и полученного будет промежуток .

  3. Если , то

Поскольку в результате преобразований получили верное неравенство, то решением будет любое действительное значение переменной: . Пересекаем с промежутком, на котором рассматриваем, и в результате получаем, что .

Объединяя полученные интервалы в случаях 1-3, запишем решение заданного неравенства:

   

Ответ
Понравился сайт? Расскажи друзьям!

Неравенства с модулем.

Способы решения неравенств с модулями Неравенства с модулем

2. Способы решения неравенств с модулями:

2
1. По определению модуля
2. Возведение обоих частей неравенства
в квадрат
3. Замена переменной
4. Раскрытие модуля на промежутке
знакопостоянства
5. Равносильность неравенств системам
6. Важный частный случай

3. 1.По определению модуля

3
| f (x) |
-a
|3x-1|
-7
-6
8
-2
3
8
Ответ: 2;
3
| f (x) |> а
a
5x 2 4
-a
a
5 x 2 4
5 x 2 4
5 x 6
5 x 2
2 6
Ответ : ; ;
5 5

4. 2.Возведение обеих частей в квадрат

4
|x2-1| > |x2-x|
(x2-1)2 > (x2-x)2 — равносильность не нарушена
(x2-1+ x2-x)(x2-1-x2+x) > 0 – разность квадратов
(2×2-x-1)(x-1) > 0

+
1
2
+
1

5. 3.Замена переменной

5
+
-2
0

+
3
t

6.

4. Раскрытие модуля на промежутках знакопостоянства 6
|x-1| + |2-x| > 3
x-1

2-x
+
1
+
Нули подмодульных выражений: x =1 и x =2
2
+
+

а)
б)
в)
x 1
( x 1) 2 x 3
x 1
x 0
1 x 2
x 1 x 3
x 2
x 3 x 3
x 2
x 3
0
1
х ;0
1 x 2
1 3 неверное
Ответ : ;0 3;
2
3
х 3;

7. 5. Равносильность неравенств системам или их совокупности

7
См. решение по определению
Равносильно неравенству:
Можно записать в виде
системы
Неравенство равносильно двум
неравенствам:
Можно записать в виде совокупности

8. 5. Равносильность неравенств системам (примеры)

8
№1
3x | 2 x | 5
№2
5 x 7 | x 2 |
| 2 x | 5 3x
| x 2 | 5 x 7
2 x 5 3 x
2 x 3 x 5
1
x 1 2
x 1 3
4
x 2 5x 7
x 2 7 5x
1
x 2 4
x 5
6
1
Ответ : ( ;1 ]
2
1
Ответ : ( ;2 )
4

9.

6. Один частный случай 9
x 1
1
x 2
x 1
x 2
ОДЗ : x 2
1
умножим на |x+2|>0 в ОДЗ
| x 1 | | x 2 |
возведем в квадрат, обе части
( x 1 x 2)( x 1 x 2) 0
(2 x 1)( 3) 0
2x 1 0
x 12
для преобразования используем
разность квадратов
Учитывая ОДЗ, получим:
1
Ответ : ( ; 2) ( 2; )
2
Обучающая самостоятельная работа
10
Метод решения
1. По определению модуля
условие
ответы
(-5; 1)
По определению модуля
По определению модуля
По определению модуля
(-∞; −2) ∪ (−2; −0,5)
2. Возведение обеих частей в
квадрат
3. Раскрытие модуля на
промежутках
знакопостоянства
4. Замена переменной
Замена переменной
5. Замена совокупностью
систем
0; 2
11

Решение системы неравенств с модулем

Решим систему неравенств с модулем из варианта №50 А. Ларина.

Решим каждое неравенство системы по отдельности, а потом совместим решения обоих неравенств на одной координатной прямой.

1. Решим первое неравенство системы.

Чтобы решить неравенство, содержащее модули, нужно раскрыть модули.

Приравняем каждое подмодульное выражение к нулю и найдем точки, в которых подмодульные выражения меняют знак.

Нанесем эти значения на числовую прямую:

Мы получили три промежутка. Найдем знаки каждого подмодульного выражения на каждом промежутке:

Раскроем модули на каждом промежутке (мы можем граничные точки  и  включать в оба промежутка):

а)

На этом промежутке оба подмодульных выражения отрицательны, поэтому мы раскрываем модули с противоположным знаком:

(1)

Так как исходное неравенство «превращается» в неравенство (1) только при , получим систему неравенств:

.

Решим первое неравенство, и получим систему:

.

Решением системы неравенств является промежуток:

б)

На этом промежутке первое подмодульное выражение положительно, а второе отрицательно, поэтому первый модуль мы раскрываем с тем же знаком, а второй с противоположным.

Получаем неравенство:

(2)

Так как исходное неравенство «превращается» в неравенство (2) только при , получим систему неравенств:

или

Решением системы неравенств является промежуток:

в)

На этом промежутке оба подмодульных выражения положительны, поэтому оба модуля мы раскрываем с тем же знаком.

Получаем неравенство:

(3)

Так как исходное неравенство «превращается» в неравенство (3) только при , получим систему неравенств:

или

Решением системы является промежуток:

Объединим три промежутка и получим решение первого неравенства исходной системы:

2. Решим второе неравенство системы.

Приведем левую часть неравенства к общему основанию. Сначала разложим на множители знаменатель первой дроби:

Решим это неравенство методом интервалов.

Найдем корни числителя и знаменателя и нанесем их на числовую ось.

На самом правом промежутке , поэтому знаки расставим так:

Нас интересуют промежутки со знаком «-«:

следовательно, решение этого неравенства:

Совместим решения первого и второго неравенств исходной системы на одной координатной прямой и найдем их пересечение:

 

Ответ: [-2;1)(2;2,4]


И.В. Фельдман, репетитор по математике.

Неравенства с модулем

Существует несколько способов решения неравенств, содержащих модуль. Рассмотрим некоторые из них.

1) Решение неравенства с помощью геометрического свойства модуля.

Напомню, что такое геометрическое свойство модуля: модуль числа x – это расстояние от начала координат до точки с координатой x.

В ходе решения неравенств этим способом может возникнуть 2 случая:

1. |x| ≤ b, тогда картинка решения выглядит так:

И неравенство с модулем очевидно сводится к системе двух неравенств. Тут знак может быть и строгим, в этом случае точки на картинке будут «выколотыми».

2. |x| ≥ b, тогда картинка решения выглядит так:

И неравенство с модулем очевидно сводится к совокупности двух неравенств. Тут знак может быть и строгим, в этом случае точки на картинке будут «выколотыми».

Пример 1.

Решить неравенство |4 – |x|| 3.

Решение.

Данное неравенство равносильно следующей совокупности:

[4 – |x| ≤ -3
[4 – |x| ≥ 3.

Хочу напомнить принципиальное отличие понятия совокупности от понятия системы. Когда мы ставим знак системы « { », мы подразумеваем, что выполняются и первое и второе неравенства одновременно, то есть мы ищем общие решения двух неравенств. Когда мы ставим знак совокупности « [ », мы подразумеваем, что выполняется или первое неравенство, или второе, то есть мы ищем те значения неизвестного x, которые являются решением либо первого, либо второго неравенства.

Теперь решаем систему.

[-|x| ≤ -7
[-|x| ≥ -1,
[|x| ≥ 7
[|x| ≤ 1.

Решаем отдельно первое неравенство:

[x ≥ 7
[x ≤ -7.

Решаем отдельно второе неравенство:

{x ≥ -1
{x ≤ 1.

Мы получили совокупность, состоящую из подсовокупности и системы. Решением исходного неравенства будут все x, которые удовлетворяют хотя бы одному неравенству из совокупности и каждому из неравенств системы.

Ответ: x € (-∞; -7] U [-1;1] U [7; +∞]

Пример 2.

Решить неравенство ||x+2| – 3| 2.

Решение.

Данное неравенство равносильно следующей системе.

{|x + 2| – 3 ≥ -2
{|x + 2| – 3 ≤ 2,
{|x + 2| ≥ 1
{|x + 2| ≤ 5.

Решим отдельно первое неравенство системы. Оно эквивалентно следующей совокупности:

[x + 2 ≥ 1
[x + 2 ≤ -1,
[x ≥ -1
[x ≤ -3.

Решим отдельно второе неравенство системы. Оно эквивалентно следующей системе:

{x + 2 ≤ 5
{x + 2 ≥ -5,
{x ≤ 3
{x ≥ -7.

Мы получили систему, состоящую из подсистемы и совокупности. Решением исходного неравенства будут все x, которые являются одновременно решением совокупности и решением подсистемы.

Ответ: х € [-7; -3] U [-1; 3].

2) Решение неравенств, используя определение модуля.

Напомню для начала определение модуля.

|a| = a, если a 0 и |a| = -a, если a < 0.

Например, |34| = 34, |-21| = -(-21) = 21.

Пример 1.

Решить неравенство 3|x – 1| x + 3.

Решение.

Используя определение модуля получим две системы:

{x – 1 ≥ 0
{3(x – 1) ≤ x + 3

и

{x – 1 < 0
{-3(x – 1) ≤ x + 3.

Решая первую вторую системы в отдельности, получим:

{x ≥ 1
{x ≤ 3,

{x < 1
{x ≥ 0.

Решением исходного неравенства будут все решения первой системы и все решения второй системы.

Ответ: x € [0; 3].

3) Решение неравенств методом возведения в квадрат.

Пример 1.

Решить неравенство |x2 – 1| < | x2 – x + 1|.

Решение.

Возведем обе части неравенства в квадрат. Замечу, что возводить обе части неравенства в квадрат можно только в том случае, когда они обе положительные. В данном случае у нас и слева и справа стоят модули, поэтому мы можем это сделать.

(|x2 – 1|)2 < (|x2 – x + 1|)2.

Теперь воспользуемся следующим свойством модуля: (|x|)2 = x2.

(x2 – 1)2 < (x2 – x + 1)2,

(x2 – 1)2 – (x2 – x + 1)2 < 0.

Дальше лучше всего воспользоваться формулой разности квадратов. Можно, конечно и возводить в квадрат левую и правую скобку, но это займет гораздо больше времени.

(x2 – 1 – x2 + x – 1)( x2 – 1 + x2 – x + 1) < 0,

(x – 2)(2x2 – x) < 0,

x(x – 2)(2x – 1) < 0.

Решаем методом интервалов.

Ответ: x € (-∞; 0) U (1/2; 2)

4) Решение неравенств методом замены переменных.

Пример.

Решить неравенство (2x + 3)2 – |2x + 3| 30.

Решение.

Заметим, что (2x + 3)2 = (|2x + 3|)2. Тогда получим неравенство

(|2x + 3|)2 – |2x + 3| ≤ 30.

Сделаем замену y = |2x + 3|.

Перепишем наше неравенство с учетом замены.

y2 – y ≤ 30,

y2 – y – 30 ≤ 0.

Разложим квадратный трехчлен, стоящий слева, на множители.

D = 121,

y1 = (1 + 11) / 2,

y2 = (1 – 11) / 2,

y1 = 6,

y2 = -5.

(y – 6)(y + 5) ≤ 0.

Решим методом интервалов и получим:

-5 ≤ y ≤ 6.

Вернемся к замене:

-5 ≤ |2x + 3| ≤ 6.

Данное двойное неравенство равносильно системе неравенств:

{|2x + 3| ≤ 6
{|2x + 3| ≥ -5.

Решим каждое из неравенств в отдельности.

Первое равносильно системе

{2x + 3 ≤ 6
{2x + 3 ≥ -6.

Решим ее.

{x ≤ 1.5
{x ≥ -4.5.

Второе неравенство очевидно выполняется для всех x, так как модуль по определению число положительное. Так как решение системы – это все x, которые удовлетворяют одновременно и первому и второму неравенству системы, то решением исходной системы будет решение ее первого двойного неравенства (ведь второе верно для всех x).

Ответ: x € [-4,5; 1,5].

© blog.tutoronline.ru, при полном или частичном копировании материала ссылка на первоисточник обязательна.

Рациональные неравенства — Математика — Теория, тесты, формулы и задачи

Оглавление:

 

Основные теоретические сведения

Некоторые рекомендации к решению рациональных неравенств

К оглавлению. ..

При решении линейных неравенств есть только одна большая фишка: необходимо менять знак неравенства при делении (или умножении) неравенства на отрицательное число. Менять знак неравенства значит изменять знак «меньше» на знак «больше» или наоборот. При этом знаки плюс на минус в обход ранее изученных математических правил нигде менять не надо. Если мы делим или умножаем неравенство на положительное число знак неравенства менять не нужно. В остальном решение линейных неравенств полностью идентично решению линейных уравнений.

В линейных и в любых других рациональных неравенствах ни в коем случае нельзя домножать или делить левую или правую части неравенства на выражения, содержащие переменную (кроме случаев, когда данное выражение положительно либо отрицательно на всей числовой оси, в этом случае при делении на всегда отрицательное выражение знак неравенства нужно поменять, а при делении на всегда положительное выражение знак неравенства нужно сохранить).

Решение неравенств вида:

Проводится с помощью метода интервалов, который состоит в следующем:

  1. Изображаем координатную прямую, на которую наносим все числа ai. Эти числа, расположенные в порядке возрастания, разобьют координатную прямую на (n+1) промежутков знакопостоянства функции f(x).
  2. Таким образом, определив знак f(x) в любой точке каждого промежутка (обычно эта точка выбирается из удобства арифметических действий), определяем знак функции на каждом промежутке. Главное при этом не подставлять в функцию сами границы промежутков.
  3. Выписываем в ответ все те промежутки, знак функции на которых соответствуют основному условию неравенства.

Нужно также отметить, что не обязательно исследовать знак функции на каждом промежутке подстановкой некоторого значения из этого промежутка. Достаточно определить таким образом знак функции только на одном промежутке (обычно на крайнем правом), а затем двигаясь от этого промежутка влево вдоль числовой оси можно чередовать знаки промежутков по принципу:

  • Если скобка из которой взялось число через которое мы переходим стоит в нечетной степени, то при переходе через соответствующую точку знак неравенства меняется.
  • А если соответствующая скобка стоит в четной степени, то при переходе через соответствующую точку знак неравенства не меняется.

При этом нужно учитывать еще и следующие замечания:

  • В строгих неравенствах (знаки «меньше» или «больше») границы промежутков никогда не входят в ответ, а на числовой оси они изображаются выколотыми точками.
  • В нестрогих неравенствах (знаки «меньше либо равно» или «больше либо равно») те границы промежутков, которые взяты из числителя всегда входят в ответ и изображаются закрашенными точками (так как в этих точках функция действительно обращается в ноль, что удовлетворяет условию).
  • А вот границы взятые из знаменателя в нестрогих неравенствах всегда изображаются выколотыми точками и в ответ никогда не входят (так как в этих точках в ноль обращается знаменатель, что недопустимо).
  • Во всех неравенствах если одна и та же скобка есть и в числителе и в знаменателе, то сокращать на эту скобку нельзя. Нужно изобразить соответствующую ей точку выколотой на оси, и не забыть исключить из ответа. При этом при чередовании знаков промежутков, проходя через эту точку знак менять не нужно.

Итак еще раз самое важное: при записи окончательного ответа в неравенствах не потеряйте отдельные точки, удовлетворяющие неравенству (это корни числителя в нестрогих неравенствах), и не забудьте исключить из ответа все корни знаменателя во всех неравенствах.

При решении рациональных неравенств более сложного вида чем указан выше, необходимо сначала алгебраическими преобразованиями свести их именно к такому виду, а затем применить метод интервалов с учетом всех уже описанных тонкостей. Таким образом, можно предложить следующий алгоритм для решения рациональных неравенств:

  1. Все слагаемые, дроби и другие выражения необходимо перенести в левую часть неравенства.
  2. При необходимости привести дроби к общему знаменателю.
  3. Разложить числитель и знаменатель полученной дроби на множители.
  4. Решить полученное неравенство методом интервалов.

При этом при решении рациональных неравенств не допускается:

  1. Перемножать дроби «крест-накрест».
  2. Как и в уравнениях, нельзя сокращать множители с переменной с обеих сторон неравенства. Если такие множители есть, то после переноса всех выражений в левую часть неравенства их нужно вынести за скобки, а затем учесть те точки которые они дадут после окончательного разложения полученного выражения на множители.
  3. Отдельно рассматривать числитель и знаменатель дроби.

Как и в остальных темах по математике, при решении рациональных неравенств можно применять метод замены переменной. Главное не забывать, что после введения замены, новое выражение должно стать проще и не содержать старой переменной. Кроме того, нужно не забывать выполнять обратную замену.

При решении систем рациональных неравенств нужно по очереди решить все неравенства входящие в систему.  Система требует выполнения двух и более условий, причем мы ищем те значения неизвестной величины, которые удовлетворяют сразу всем условиям. Поэтому, в ответе системы неравенств нужно указать общие части всех решений отдельных неравенств (или общие части всех заштрихованных промежутков, изображающих ответы каждого отдельного неравенства).

При решении совокупностей рациональных неравенств также по очереди решают каждое из неравенств. Совокупность требует нахождения всех значений переменной, удовлетворяющих хотя бы одному из условий. То есть любому из условий, нескольким условиям или всем условиям вместе. В ответе совокупности неравенств указывают все части всех решений отдельных неравенств (или все части всех заштрихованных промежутков, изображающих ответы каждого отдельного неравенства).

 

Решение некоторых типов неравенств с модулями

К оглавлению…

Неравенства с модулями можно и нужно решать последовательно раскрывая модули на промежутках их знакопостоянства. Таким образом, нужно поступать примерно также как при решении уравнений с модулями (об этом ниже). Но есть несколько относительно простых случаев в которых решение неравенства с модулем сводится к более простому алгоритму. Так например, решение неравенства вида:

Сводится к решению системы:

В частности неравенство:

Может быть заменено равносильной системой:

Ну а если в аналогичном неравенстве заменить знак «меньше» на «больше»:

То его решение сводится уже к решению совокупности:

В частности неравенство:

Может быть заменено равносильной совокупностью:

Таким образом, необходимо запомнить, что для неравенства «модуль меньше» мы получаем систему, где должны одновременно выполняться оба условия, а для неравенства «модуль больше» мы получаем совокупность, в которой должно выполняться любое из условий.

При решении рациональных неравенств с модулем вида:

Целесообразно переходить к следующему равносильному рациональному неравенству без модуля:

Такое неравенство нельзя решать извлечением корня (если по-честному извлекать корень, то снова нужно поставить модули, и Вы вернетесь к началу, если про модули забыть, это равносильно тому, чтобы в самом начале про них просто забыть, а это, конечно, ошибка). Все скобки нужно перенести налево и, ни в коем случае не раскрывая скобки, применить формулу разности квадратов.

Еще раз повторимся, что для решения всех других типов неравенств с модулями кроме указанных выше нужно раскрывать все модули входящие в неравенство на промежутках их знакопостоянства и решать полученные неравенства. Напомним подробнее общий смысл этого алгоритма:

  • Сначала находим точки на числовой оси, в которых обращается в ноль каждое из выражений, стоящих под модулем.
  • Далее делим всю числовую ось на интервалы между полученными точками и исследуем знак каждого из подмодульных выражений на каждом интервале. Заметьте, что для определения знака выражения надо подставить в него любое значение переменной из интервала, кроме граничных точек. Выбирайте те значения переменной, которые легко подставлять.
  • Далее на каждом полученном интервале раскрываем все модули в исходном неравенстве в соответствии с их знаками на данном интервале и решаем полученное обычное рациональное неравенство с учетом всех правил и тонкостей решения обычных неравенств без модулей.
  • Решение каждого из неравенств полученных на конкретном промежутке объединяем в систему с самим промежутком, а все такие системы объединяем в совокупность. Таким образом из решений всех неравенств выбираем только те части которые вошли в промежуток, на котором было получено данное неравенство, и записываем все эти части в итоговый ответ.

3.2.4. Неравенства с модулем



Глава 3. Решение уравнений и неравенств

3.2.

3.2.4.

Основные способы решений неравенств с модулем во многом совпадают с методами решения аналогичных уравнений. Единственное отличие, пожалуй, связано с тем, что, решая неравенства с модулем (как, впрочем, и неравенства вообще), нужно очень внимательно совершать равносильные переходы и следить не только за тем, чтобы не приобрести новые решения, но и за тем, чтобы не потерять уже имеющиеся.

Стандартный путь решения неравенств с модулем заключается в том, что координатная прямая разбивается на промежутки (границами этих промежутков являются нули подмодульных выражений), а затем неравенство решается на каждом из промежутков.

Этот метод работает всегда. Правда, в отдельных случаях может быть затруднена его техническая реализация, например, очень тяжело или невозможно найти корни подмодульных выражений и пр. Однако, это сложности иного плана. Нужно понимать, что раскрытие модуля по определению неизменно приводит к цели. Конечно же, этот метод не является оптимальным: в условиях конкурсного экзамена важен не только результат, но и то время, которое потрачено на его получение.

Рассмотрим методы, не связанные с поиском нулей функций, стоящих под знаком модуля.

Рассмотрим неравенство Очевидно, что те x, для которых g (x) < 0, не являются решениями. Значит, если x является решением, то для него g (x) ≥ 0, и согласно геометрическому смыслу модуля, как расстоянию на координатной оси, данное неравенство равносильно системе Таким образом, имеем

Аналогично можно рассмотреть неравенство Неравенство выполнено для тех x, для которых g (x) < 0 и функции f (x) и g (x) определены. Для тех x, для которых g (x) ≥ 0, имеем равносильную совокупность


Заметим, что последняя совокупность является равносильной нашему неравенству и при g (x) ≤ 0. В этом можно непосредственно убедиться, учтя g (x) ≤ 0 и вспомнив определение знака совокупности.

Как видно, в простых случаях особых преимуществ метод перехода к равносильной системе не имеет, но иногда его преимущества весьма заметны.

Пример 2

Решите неравенство

Как видно, найти значения x, при которых подмодульное выражение обращается в нуль, чрезвычайно затруднительно. Однако переход к равносильной системе значительно упрощает дело. Имеем:


Ответ.  






Алгебра — абсолютные неравенства

Показать мобильное уведомление Показать все примечания Скрыть все примечания

Похоже, вы находитесь на устройстве с «узкой» шириной экрана ( т.е. вы, вероятно, на мобильном телефоне). Из-за характера математики на этом сайте лучше всего просматривать в ландшафтном режиме.Если ваше устройство не находится в ландшафтном режиме, многие уравнения будут отображаться сбоку вашего устройства (должна быть возможность прокрутки, чтобы увидеть их), а некоторые пункты меню будут обрезаны из-за узкой ширины экрана.

Раздел 2-15: Неравенства абсолютного значения

В предыдущем разделе мы решали уравнения, содержащие абсолютные значения. В этом разделе мы хотим рассмотреть неравенства, содержащие абсолютные значения.Нам нужно будет рассмотреть два отдельных случая.

Неравенства с участием
< и \( \le \)

Как и в случае с уравнениями, давайте начнем с довольно простого случая.

\[\слева| р \право| \ле 4\]

Это говорит о том, что каким бы ни было \(p\), оно должно находиться на расстоянии не более 4 от начала координат. Это означает, что \(p\) должно быть где-то в диапазоне

. \[ — 4 \le p \le 4\]

Мы могли бы иметь подобное неравенство с < и получить аналогичный результат.

В общем, здесь можно использовать следующие формулы:

\[\require{bbox} \bbox[2pt,border:1px сплошной черный]{\begin{align*}{\mbox{If}}\left| р \право| \le b,\,\,\,b > 0\,\,\,\,\,\,\,\,\,\,\,{\mbox{тогда}} — b \le p \le b \\ {\mbox{Если}}\влево| р \право| 0\,\,\,\,\,\,\,\,\,\,\,{\mbox{then}} — b

Обратите внимание, что это требует, чтобы \(b\) было положительным, как мы сделали с уравнениями.

Давайте рассмотрим пару примеров.

Пример 1 Решите каждую из следующих задач.
  1. \(\влево| {2x — 4} \вправо| < 10\)
  2. \(\влево| {9m + 2} \вправо| \le 1\)
  3. \(\влево| {3 — 2z} \вправо| \le 5\)
Показать все решения Скрыть все решения a \(\left| {2x — 4} \right| Показать решение

На самом деле больше нечего делать, кроме как подключиться к формуле. Как и в случае с уравнениями, \(p\) просто представляет все, что находится внутри полос абсолютного значения.Итак, с этим первым у нас есть

\[ — 10

Теперь это не более чем довольно простое двойное неравенство, которое нужно решить, так что давайте сделаем это.

\[\begin{array}{c} — 6

Обозначение интервала для этого решения: \(\left( { — 3,7} \right)\).


b \(\left| {9m + 2} \right| \le 1\) Показать решение

Здесь особо нечего делать.

\[\begin{array}{c} — 1 \le 9m + 2 \le 1\\ — 3 \le 9m \le — 1\\ \displaystyle — \frac{1}{3} \le m \le — \frac{1}{9}\end{массив}\]

Обозначение интервала: \(\left[ { — \frac{1}{3}, — \frac{1}{9}} \right]\).


c \(\left| {3 — 2z} \right| \le 5\) Показать решение

Нам нужно быть немного осторожными с решением двойного неравенства с этим, но в остальном он почти идентичен предыдущим двум частям.

\[\begin{array}{c} -5 \le 3 — 2z \le 5\\ — 8 \le — 2z \le 2\\ 4 \ge z \ge — 1\end{array}\]

На последнем шаге не забудьте поменять направление неравенств, так как мы все поделили на отрицательное число.Обозначение интервала для этого решения: \(\left[ { — 1,4} \right]\).

Неравенства с участием > и \( \ge \)

Еще раз давайте начнем с простого примера числа.

\[\слева| р \право| \ge 4\]

Это говорит о том, что каким бы ни было \(p\), оно должно находиться на расстоянии не менее 4 от начала координат, и поэтому \(p\) должно находиться в одном из следующих двух диапазонов:

\[p \le — 4\hspace{0,25 дюйма}{\mbox{или}}\hspace{0. 25в}п\ге 4\]

Перед тем, как дать общее решение, мы должны указать на распространенную ошибку, которую учащиеся совершают при решении подобных задач. Многие студенты пытаются объединить их в одно двойное неравенство следующим образом:

\[ — 4 \ge p \ge 4\]

Хотя это может показаться логичным, мы не можем не подчеркнуть, что ЭТО НЕПРАВИЛЬНО!! Вспомним, о чем говорит двойное неравенство. В двойном неравенстве мы требуем, чтобы оба неравенства выполнялись одновременно. Приведенное выше двойное неравенство будет тогда означать, что \(p\) — это число, которое одновременно меньше -4 и больше 4.Это просто не имеет смысла. Нет числа, удовлетворяющего этому.

Эти решения должны быть записаны в виде двух неравенств.

Вот общая формула для них.

\[\require{bbox} \bbox[2pt,border:1px сплошной черный]{\begin{align*}{\mbox{If}}\left| р \право| \ge b,\,\,\,b > 0\,\,\,\,\,\,\,\,\,\,\,{\mbox{тогда}}p \le — b{\mbox {или}}p \ge b\\ {\mbox{If}}\left| р \право| > b,\,\,\,b > 0\,\,\,\,\,\,\,\,\,\,\,{\mbox{тогда}}pb\end{align*}}\ ]

Опять же, потребуем , чтобы \(b\) было здесь положительным числом. Давайте поработаем над парой примеров.

Пример 2 Решите каждую из следующих задач.
  1. \(\влево| {2x — 3} \вправо| > 7\)
  2. \(\влево| {6t + 10} \вправо| \ge 3\)
  3. \(\влево| {2 — 6л} \вправо| > 10\)
Показать все решения Скрыть все решения a \(\left| {2x — 3} \right| > 7\) Показать решение

Опять же, \(p\) представляет собой количество внутри столбцов абсолютного значения, поэтому все, что нам нужно сделать здесь, это подключиться к формуле, а затем решить два линейных неравенства.

\[\begin{align*}2x — 3 и 7\\ 2x & 10\\ x & 5\end{align*}\]

Обозначение интервала для них: \(\left( { — \infty , — 2} \right)\) или \(\left( {5,\infty } \right)\).


b \(\left| {6t + 10} \right| \ge 3\) Показать решение

Давайте просто подключимся к формулам и идем сюда,

\[\begin{align*}6t + 10 & \le — 3 & \hspace{0. 25in} & {\mbox{or}} & \hspace{0.25 дюймов} 6t + 10 & \ge 3\\ 6t & \le — 13 & \hspace{0.25in} & {\mbox{or}} & \hspace{0.25in} 6t & \ge — 7\\ t & \ le — \frac{{13}}{6} & \hspace{0.25in} & {\mbox{or}} & \hspace{0.25in}t & \ge — \frac{7}{6}\end{ выровнять*}\]

Обозначение интервала для них: \(\left( { — \infty , — \frac{{13}}{6}} \right]\) или \(\left[ { — \frac{7}{6} ,\infty } \справа)\).


c \(\left| {2 — 6y} \right| > 10\) Показать решение

Опять же, здесь особо нечего делать.

\[\begin{align*}2 — 6y & 10\\ — 6y & 8\\ y & > 2 & \hspace{0.25in} & {\mbox{or}} & \hspace{0.25in} y &

Обратите внимание, что нам пришлось изменить направление неравенства, когда мы делили на отрицательное число! Обозначение интервала для этих решений: \(\left( {2,\infty } \right)\) или \(\left( { — \infty , — \frac{4}{3}} \right)\).

Хорошо, теперь нам нужно быстро взглянуть на то, что происходит, если \(b\) равно нулю или отрицательно. Мы сделаем это с набором примеров и начнем с нуля.

Пример 3 Решите каждую из следующих задач.
  1. \(\влево| {3x + 2} \вправо| < 0\)
  2. \(\влево| {x — 9} \вправо| \le 0\)
  3. \(\влево| {2x — 4} \вправо| \ge 0\)
  4. \(\влево| {3x — 9} \вправо| > 0\)
Показать все решения Скрыть все решения Показать обсуждение

Эти четыре примера, кажется, охватывают все наши основы.


a \(\left| {3x + 2} \right| Показать решение

Теперь мы знаем, что \(\left| p \right| \ge 0\) и поэтому никогда не может быть меньше нуля. Следовательно, в этом случае решения нет, так как абсолютное значение не может быть строго меньше нуля (, т.е. отрицательно).


b \(\left| {x — 9} \right| \le 0\) Показать решение

Это почти то же самое, что и предыдущая часть. Мы по-прежнему не можем иметь абсолютное значение меньше нуля, однако оно может быть равно нулю. Таким образом, это будет иметь решение, только если

\[\слева| {х — 9} \право| = 0\]

и мы знаем, как это решить из предыдущего раздела.

\[x — 9 = 0\hspace{0,25 дюйма} \Стрелка вправо \hspace{0,25 дюйма}x = 9\]
c \(\left| {2x — 4} \right| \ge 0\) Показать решение

В этом случае давайте снова вспомним, что независимо от того, что такое \(p\), мы гарантированно имеем \(\left| p \right| \ge 0\). Это означает, что независимо от того, что такое \(x\), мы можем быть уверены, что \(\left| {2x — 4} \right| \ge 0\) будет истинным, поскольку абсолютные значения всегда будут положительными или нулевыми.

Решением в этом случае являются все действительные числа или все возможные значения \(x\). В обозначении неравенства это будет \( — \infty < x <\infty \).


d \(\left| {3x — 9} \right| > 0\) Показать решение

Этот почти идентичен предыдущей части, за исключением того, что на этот раз обратите внимание, что мы не хотим, чтобы абсолютное значение когда-либо было равно нулю. Итак, нам все равно, какое значение принимает абсолютное значение, если оно не равно нулю. Это означает, что нам просто нужно избегать значений \(x\), для которых мы получаем

\[\слева| {3x — 9} \право| = 0\hпробел{0.25 дюймов} \Стрелка вправо \hspace{0,25 дюйма} 3x — 9 = 0\hspace{0,25 дюйма} \Стрелка вправо \hspace{0,25 дюйма} x = 3\]

Решением в этом случае являются все действительные числа, кроме \(x = 3\).

Теперь давайте быстро рассмотрим примеры с отрицательными числами.

Пример 4 Постановка задачи.
  1. \(\влево| {4x + 15} \вправо| < - 2\) и \(\влево| {4x + 15} \вправо| \le - 2\)
  2. \(\left| {2x — 9} \right| \ge — 8\) и \(\left| {2x — 9} \right| > — 8\)
Показать все решения Скрыть все решения Показать обсуждение

Обратите внимание, что мы работаем над ними парами, потому что на этот раз, в отличие от предыдущего набора примеров, решения будут одинаковыми для всех.

Оба (все четыре?) из них будут использовать тот факт, что независимо от того, что такое \(p\), мы гарантированно имеем \(\left| p \right| \ge 0\). Другими словами, абсолютные значения всегда положительны или равны нулю.


a \(\left| {4x + 15} \right| Показать решение

Хорошо, если абсолютные значения всегда положительны или равны нулю, они никак не могут быть меньше или равны отрицательному числу.

Следовательно, ни для одного из них нет решения.


b \(\left| {2x — 9} \right| \ge — 8\) и \(\left| {2x — 9} \right| > — 8\) Показать решение

В этом случае, если абсолютное значение положительно или равно нулю, оно всегда будет больше или равно отрицательному числу.

Решением для каждого из них являются все действительные числа.

неравенств, связанных с абсолютными значениями | Решенные примеры | Алгебра

Абсолютное неравенство — это неравенство, в котором есть символ абсолютного значения.

Можно решить двумя способами.

  1. Использование числовой строки
  2. Использование формул

Зная эти методы, вы сможете решать задачи с абсолютными неравенствами с калькулятором абсолютных неравенств или без него.

Ознакомьтесь с интерактивными симуляциями, чтобы узнать больше об уроке, и попробуйте свои силы в решении нескольких интересных практических вопросов в конце страницы.

План урока  

Что такое абсолютные неравенства?

Неравенство с абсолютным значением – это неравенство, включающее выражение абсолютного значения с переменными.

То есть абсолютное неравенство может быть одной из следующих форм (или) может быть преобразовано в одну из следующих форм:

\(\begin{align} |ax+b| &< c\\[0,2 см] |ax+b|&>c \\[0,2 см] |ax+b| &\leq c\\ [0,2 см] |ax+b| &\geq c \end{align} \)

Итак, абсолютные неравенства бывают двух типов:

  • один с \(<\) или \(\leq\) 
  • один с \(>\) или \(\geq\)

Мы подробно изучим каждый тип неравенства.


Как решать абсолютные неравенства?

Вот процедура решения абсолютного неравенства любого типа с помощью числовой прямой.

Процедура показана шаг за шагом вместе с примером.

Пример

Решить абсолютное неравенство \(|x+2| < 4\)

Шаг 1:  Представим неравенство как уравнение и решим его.

Превратите знак неравенства «<" в нашем неравенстве в "=" и решите его.

\[|х+2|=4\]

Удалив знак абсолютного значения с левой стороны, мы получим знак \(\pm\) с другой стороны.

\[x+2=\pm 4\]

Это приводит к двум уравнениям, одно со знаком «+», другое со знаком «-».

\[ \begin{align} x+2 &=4 & x+2 &=-4\\[0.2cm] x&=2 & x&=-6 \end{align}\]

Шаг 2: Представьте решения из шага 1 на числовой прямой по порядку.

Здесь мы видим, что числовая линия разделена на 3 части.

Шаг 3: Возьмите случайное число из каждого из этих интервалов и подставьте его в данное неравенство.

Определите, какие из этих чисел действительно удовлетворяют заданному неравенству.

Интервал

Случайное число Проверка заданного неравенства случайным числом
\((-\infty, -6)\) -7 \[ \begin{align} |-7+2|<4\\[0.2см] 5<4\\[0,2см] \text{Это неверно}  \end{выравнивание}\]
\((-6, 2)\) 0 \[ \begin{align} |0+2|<4\\[0,2 см] 2<4\\[0,2 см] \text{Это правда}  \end{align}\]
\((2, \infty)\) 3 \[ \begin{align} |3+2|<4\\[0,2 см] 5<4\\[0,2 см] \text{Это неверно}  \end{align}\]

Шаг 4: Решением данного неравенства является интервал(ы), которые приводят к True в приведенной выше таблице

Следовательно, решение данного неравенства:

\((-6,2)\) (или) \(-6

Эта процедура представлена ​​на следующей блок-схеме.

Примечание:

  1.   Если бы проблема была \(|x+2| \leq 4\), то решение было бы \([-6, 2]\) (или) \(-6\leq x \leq 2 \). т. е. \[\begin{array}{l}\text { If }|x+2|<4 \Rightarrow (-6, 2) \text { ИЛИ } -6 < x <2 \\\text { If } |х+2| \leq  \Стрелка вправо [-6, 2] \text{ ИЛИ }  -6 \leq x \leq 2\end{массив}\]
  2.   Кроме того,\(\text{If} |x+2|\!>\!4\Стрелка вправо(-\infty,-6)\cup(2, \infty)\)
    \(\text{(ИЛИ)}(-\infty \(\text { If }|x+2| \geq 4 \Rightarrow( -\infty,-6] \cup[2, \infty)\)
    \(\text{ (ИЛИ) }(-\infty

Тот же трюк объясняется в следующих «Важных примечаниях».»


Графики абсолютных неравенств

При графическом отображении абсолютного неравенства мы должны помнить о следующих вещах.

  • Используйте открытые точки на концах открытых интервалов (т.  е. таких интервалов, как \((a,b\)).
  • Используйте закрытые точки на концах закрытых интервалов (например, таких интервалов, как \([a,b]\)).

 

Важные примечания

  1. Если рядом с числом стоит скобка, это означает, что это число НЕ включено в решение.
  2. Если перед числом стоит квадратная скобка, это означает, что число включено в решение.
  3. Мы всегда используем круглые скобки в точках \(-\inf\) или \(\inf\) независимо от данного неравенства.
  4. Мы решаем использовать квадратные скобки (или) круглые скобки для числа в зависимости от того, есть ли в данном неравенстве «=».

Общая формула абсолютных неравенств

Итак, мы изучили процедуру решения абсолютных неравенств с использованием числовой прямой.

Эта процедура работает для любого типа неравенства.

На самом деле, неравенства также можно решать с помощью формул.

Чтобы применить формулы, во-первых, нам нужно выделить выражение абсолютного значения в левой части неравенства.

Для решения неравенств по формулам нужно помнить 4 случая.

Предположим, что \(а\) — положительное действительное число во всех случаях.

Случай 1: когда неравенство имеет вид:

\[|x|

В этом случае для решения неравенства используем следующие формулы:

\(\ начало {массива} {l}
\text { Если }|x| \text { Если }|x| \leq a \Rightarrow-a \leq x \leq a
\конец{массив}\)

Случай 2. Когда неравенство имеет вид:

\[|x|>a \text { или }|x| \geq a \]

В этом случае для решения неравенства используем следующие формулы:

\(\ начало {массива} {l}
\text { Если }|x|>a \Rightarrow x<-a \text { или } x>a \\
\text { Если }|x| \geq a \Rightarrow x \leq-a \text { или } x \geq a
\конец{массив}\)

Случай 3. Когда неравенство имеет вид:

\[|х| <-a  \text { или }  |x| \leq-a\]

Мы знаем, что абсолютное значение всегда дает положительное значение.

Таким образом, \(|x|\) всегда положителен.

Кроме того, \(-a\) отрицательно (мы предполагали, что \(a\) положительно).

Таким образом, два приведенных выше неравенства означают, что «положительное число меньше (или меньше или равно) отрицательного числа», что никогда не бывает верным.

Таким образом, все такие неравенства не имеют решения.

\(\begin{align} |x|<-a  &\text { or } |x| \leq-a \\[0,2 см]&\Стрелка вправо \\[0,2 см] \text{ Нет } & \text{Решение} \end{выровнено}\)

Случай 4. Когда неравенство имеет вид:

\[|х| >-a  \text { или }  |x| \geq-a\]

Мы знаем, что абсолютное значение всегда дает положительное значение.

Таким образом, \(|x|\) всегда положителен.

Кроме того, \(-a\) отрицательно (мы предполагали, что \(a\) положительно).

Таким образом, два приведенных выше неравенства означают, что «положительное число больше (или больше или равно) отрицательного числа», что всегда верно.

Таким образом, решением всех таких неравенств является множество всех действительных чисел \(R\).

\(\begin{align} |x|>-a  &\text { or } |x| \geq-a \\[0,2 см]&\Rightarrow \\[0.2cm] \text{ Набор всех } & \text{ Вещественных чисел, R} \end{aligned}\)

Все эти формулы абсолютного неравенства суммированы в следующей блок-схеме.

Вы можете найти примеры абсолютного неравенства в разделе «Решенные примеры» на этой странице.

 

Советы и рекомендации

После того, как мы приведем неравенство к виду \(|x| \text{символ неравенства }a\), если \(a\) отрицательно, то решать его никаким методом не нужно.
Сразу же мы можем написать ответ, используя следующие приемы.

  1. Если символом неравенства является \(<\) или \(\leq\), то неравенство не имеет решения.
  2. Если символом неравенства является \(>\) или \(\geq\), то решением является «множество всех действительных чисел, \(R\)».

Калькулятор абсолютных значений неравенств

Вот «Калькулятор абсолютного неравенства».

Позволяет найти решение любого абсолютного неравенства и показывает решение в 3-х формах: форма неравенства, интервальная форма и графическая форма.


Решенные примеры

Вот задачи с абсолютным неравенством.

 

 

Марка просят решить следующее абсолютное неравенство и записать решение в 3-х формах.

  1. Форма неравенства
  2. Интервальная форма
  3. Графическая форма

 Можем ли мы помочь ему найти решение?

\[|2 x-1|-9 \geq-5\]

Также покажите ему решение числовой прямой.

Раствор

Данное абсолютное значение неравенства равно:

\[|2 x-1|-9 \geq-5\]

Во-первых, нам нужно изолировать выражение абсолютного значения.

Для этого добавим по 9 с обеих сторон. Тогда получаем:

\[|2 х-1| \geq 4\]

Используя формулы, которые мы выучили:

\[ \begin{align}
&2 x-1 \leq-4 \text{ (или) }2 x-1 \geq 4\\[0,2 см]
&\text { Добавление 1 с обеих сторон }\\[0,2 см]
&2 x \leq-3 \text{ (или) } 2 x \geq 5 \\[0.2см]
&\text { Деление обеих сторон на } 2,\\[0,2 см]
&x \leq-\frac{3}{2} \text{ (или) } x \geq \frac{5}{2}
\end{выравнивание} \]

Следовательно, решение:

\(x \leq-\frac{3}{2} \text{ (или) } x \geq, \frac{5}{2}\)
(ИЛИ)
\( \left(-\infty,-\frac{3}{2}\right)\cup\left(\frac{5}{2}, \infty\right) \)

Представим решение на графике.

 

 

Плотник делает стол ручной работы. Чтобы ножка подходила, она должна быть шириной 250 миллиметров, что позволяет погрешность в 3,5 миллиметра.

Напишите неравенство абсолютного значения, которое моделирует эту связь, а затем найдите диапазон ширины ножек стола.

Раствор

Предположим, что ширина ножки стола равна \(x\).

Тогда абсолютное неравенство, соответствующее данному сценарию, равно

\[|х-250| \leq 3.5\]

Используя формулы, которые мы выучили:

\[ \begin{align} &-3.5 \leq x-250 \leq 3.5\\[0.2cm] & \text{Добавление 250 со всех сторон,}\\[0.2cm] &246.5\leq x \ leq 253.5 \end{align} \]

Диапазон ширины ножки стола:

\([246,5,253,5]\) (в мм)

 

 

Прежде чем кусок стали можно будет продать по максимальной цене, он должен иметь длину 50 футов с абсолютной погрешностью 2 фута.

Найдите диапазон допустимых высот для стальных изделий, которые должны продаваться по полной цене, написав неравенство абсолютного значения, представляющее эту ситуацию, а затем решив его.

Раствор

Предположим, что высота куска стали равна \(x\) футов.

Тогда абсолютное неравенство, соответствующее данному сценарию, равно

\[|х-50| \leq 2\]

Используя формулы, которые мы выучили:

\[ \begin{align} &-2 \leq x-50 \leq 2\\[0.2cm] & \text{Добавление 50 со всех сторон,}\\[0.2cm] &48 \leq x \leq 52\end{align} \]

Диапазон допустимых высот сталей:


Практические задачи

Вот несколько упражнений для практики. Выберите/введите свой ответ и нажмите кнопку «Проверить ответ», чтобы увидеть результат.

 

 

 

 

 


Подведем итоги

Мы надеемся, что вам понравилось изучение абсолютных неравенств с помощью моделирования и практических вопросов. Теперь вы сможете легко решать задачи на абсолютные неравенства, калькулятор абсолютных неравенств и примеры абсолютных неравенств.

О Куэмате

В Cuemath наша команда экспертов по математике стремится сделать обучение интересным для наших любимых читателей, студентов!

Благодаря интерактивному и увлекательному подходу «обучение-преподавание-обучение» учителя изучают тему со всех сторон.

Будь то рабочие листы, онлайн-классы, сеансы сомнений или любая другая форма отношений, это логическое мышление и разумный подход к обучению, в которые мы в Cuemath верим.


Часто задаваемые вопросы (FAQ)

1. Где найти калькулятор абсолютных неравенств?

Вы можете найти «калькулятор абсолютных неравенств» на этой странице.

2. Как построить график абсолютного неравенства?

При построении графика абсолютного неравенства мы должны помнить о следующих вещах.

  • Используйте открытые точки на концах открытых интервалов (т.е. интервалы типа \((a,b\)).
  • Используйте закрытые точки на концах закрытых интервалов (т. е. таких интервалов, как \([a,b]\)).

Для получения дополнительной информации вы можете перейти к разделу «Построение графиков абсолютных неравенств» на этой странице.

Неравенства абсолютного значения — Алгебра 1

Если вы считаете, что контент, доступный с помощью Веб-сайта (как это определено в наших Условиях обслуживания), нарушает одно или более ваших авторских прав, пожалуйста, сообщите нам, предоставив письменное уведомление («Уведомление о нарушении»), содержащее в информацию, описанную ниже, назначенному агенту, указанному ниже.Если университетские наставники примут меры в ответ на ан Уведомление о нарушении, он предпримет добросовестную попытку связаться со стороной, предоставившей такой контент средства самого последнего адреса электронной почты, если таковой имеется, предоставленного такой стороной Varsity Tutors.

Ваше Уведомление о нарушении может быть направлено стороне, предоставившей контент, или третьим лицам, таким как в виде ChillingEffects.org.

Обратите внимание, что вы будете нести ответственность за ущерб (включая расходы и гонорары адвокатов), если вы существенно искажать информацию о том, что продукт или деятельность нарушают ваши авторские права.Таким образом, если вы не уверены, что содержимое находится на Веб-сайте или на который ссылается Веб-сайт, нарушает ваши авторские права, вам следует сначала обратиться к адвокату.

Чтобы подать уведомление, выполните следующие действия:

Вы должны включить следующее:

Физическая или электронная подпись владельца авторских прав или лица, уполномоченного действовать от его имени; Идентификация авторских прав, которые, как утверждается, были нарушены; Описание характера и точного местонахождения контента, который, как вы утверждаете, нарушает ваши авторские права, в \ достаточно подробно, чтобы преподаватели университета могли найти и точно идентифицировать этот контент; например, мы требуем а ссылку на конкретный вопрос (а не только название вопроса), который содержит содержание и описание к какой конкретной части вопроса — изображению, ссылке, тексту и т. д. — относится ваша жалоба; Ваше имя, адрес, номер телефона и адрес электронной почты; и Заявление от вас: (а) что вы добросовестно полагаете, что использование контента, который, как вы утверждаете, нарушает ваши авторские права не разрешены законом или владельцем авторских прав или его агентом; б) что все информация, содержащаяся в вашем Уведомлении о нарушении, является точной, и (c) под страхом наказания за лжесвидетельство вы либо владельцем авторских прав, либо лицом, уполномоченным действовать от их имени.

Отправьте жалобу нашему назначенному агенту по адресу:

Чарльз Кон Varsity Tutors LLC
101 S. Hanley Rd, Suite 300
Сент-Луис, Миссури 63105

Или заполните форму ниже:

 

2.8: Решение абсолютных неравенств

Готовясь к решению уравнений абсолютного значения, мы пересматриваем наше определение абсолютного значения .

АБСОЛЮТНОЕ ЗНАЧЕНИЕ

Абсолютное значение числа — это его расстояние от нуля на числовой прямой.

Абсолютное значение числа n записывается как \(|n|\) и \(|n|\geq 0\) для всех чисел.

Абсолютные значения всегда больше или равны нулю.

Мы узнали, что и число, и его противоположность находятся на одинаковом расстоянии от нуля на числовой прямой. Поскольку они имеют одинаковое расстояние от нуля, они имеют одинаковое абсолютное значение. Например:

  • \(−5\) находится на расстоянии 5 единиц от 0, поэтому \(|−5|=5\).
  • \(5\) находится на расстоянии 5 единиц от 0, поэтому \(|5|=5\).

Рисунок \(\PageIndex{1}\) иллюстрирует эту идею.

Рисунок \(\PageIndex{1}\): числа 5 и \(−5\) отстоят от нуля на пять единиц.

Для уравнения |x|=5,|x|=5 мы ищем все числа, которые делают это утверждение верным. Мы ищем числа, расстояние которых от нуля равно 5. Мы только что видели, что и 5, и −5−5 — это пять единиц от нуля на числовой прямой. Они являются решениями уравнения.

\(\ begin{array} {ll} {\text{If}} &{|x|=5} \\ {\text{then}} &{x=−5\text{or}x=5} \\ \конец{массив}\)

Решение можно упростить до одного оператора, написав \(x=\pm 5\).Это читается так: « x равно положительным или отрицательным 5».

Мы можем обобщить это до следующего свойства для уравнений абсолютного значения.

АБСОЛЮТНЫЕ УРАВНЕНИЯ

Для любого алгебраического выражения u и любого положительного действительного числа a ,

\[\begin{array} {ll} {\text{if}} &{|u|=a} \\ {\text{then}} &{u=-a \text{или}u=a} \\ \номер \конец{массив}\]

Помните, что абсолютное значение не может быть отрицательным числом.

Пример \(\PageIndex{1}\)

Решить:

  1. \(|х|=8\)
  2. \(|у|=−6\)
  3. \(|z|=0\)
Ответить на

\(\begin{array} {ll} {} &{|x|=8} \\ {\text{Запишите эквивалентные уравнения. }} &{x=−8 \text{или } x=8} \ \ {} &{x=\pn 8} \\ \end{массив}\)

Ответ b

\(\begin{array} {ll} {} &{|y|=−6} \\ {} &{\text{Нет решения}} \\ \end{массив}\)
Поскольку абсолютное значение всегда положителен, у этого уравнения нет решений.

Ответ c

\(\begin{array} {ll} {} &{|z|=0} \\ {\text{Запишите эквивалентные уравнения.}} &{z=−0\text{или}z=0} \ \ {\text{Поскольку }−0=0,} &{z=0} \\ \end{массив}\)
Оба уравнения говорят нам, что z=0z=0, поэтому существует только одно решение.

Пример \(\PageIndex{2}\)

Решить:

  1. \(|х|=2\)
  2. \(|у|=−4\)
  3. \(|z|=0\)
Ответить на

\(\pm 2\)

Ответ b

нет решения

Ответ c

0

Пример \(\PageIndex{3}\)

Решить:

  1. \(|х|=11\)
  2. \(|у|=−5\)
  3. \(|z|=0\)
Ответить на

\(\pm 11\)

Ответ b

нет решения

Ответ c

0

Чтобы решить уравнение абсолютного значения , мы сначала изолируем выражение абсолютного значения, используя те же процедуры, которые мы использовали для решения линейных уравнений. Как только мы изолируем выражение абсолютного значения, мы перепишем его как два эквивалентных уравнения.

Пример \(\PageIndex{4}\)

Решите \(|5x−4|−3=8\).

Ответить

Пример \(\PageIndex{5}\)

Решите: \(|3x−5|−1=6\).

Ответить

\(x=4, \space x=−\frac{2}{3}\)

Пример \(\PageIndex{6}\)

Решите: \(|4x−3|−5=2\).

Ответить

\(x=−1,\space x=\frac{5}{2}\)

Здесь приведены шаги для решения уравнения абсолютного значения.

РЕШИТЬ АБСОЛЮТНЫЕ УРАВНЕНИЯ.

  1. Изолировать выражение абсолютного значения.
  2. Напишите эквивалентные уравнения.
  3. Решите каждое уравнение.
  4. Проверьте каждое решение.

Пример \(\PageIndex{7}\)

Решите \(2|x−7|+5=9\).

Ответить
«>
\(2|x−7|+5=9\)
Изолировать выражение абсолютного значения. \(2|x−7|=4\)
\(|x−7|=2\)
Напишите эквивалентные уравнения. \(x−7=−2\) или \(x−7=2\)
Решите каждое уравнение. \(х=5\) или \(х=9\)
Чек:

Пример \(\PageIndex{8}\)

Решите: \(3|x−4|−4=8\).

Ответить

\(х=8,\пробел х=0\)

Пример \(\PageIndex{9}\)

Решите: \(2|x−5|+3=9\).

Ответить

\(х=8,\пробел х=2\)

Помните, абсолютное значение всегда положительное!

Пример \(\PageIndex{10}\)

Решите: \(|\frac{2}{3}x−4|+11=3\).

Ответить

\(\begin{array} {ll} {} &{|\frac{2}{3}x−4|=−8} \\ {\text{Изолировать термин абсолютного значения.}} &{|\ frac{2}{3}x−4|=−8} \\ {\text{Абсолютное значение не может быть отрицательным.}} &{\text{Нет решения}} \\ \end{массив}\)

Пример \(\PageIndex{11}\)

Решите: \(|\frac{3}{4}x−5|+9=4\).

Ответить

Нет решения

Пример \(\PageIndex{12}\)

Решите: \(|\frac{5}{6}x+3|+8=6\).

Ответить

Нет решения

Некоторые из наших уравнений абсолютного значения могут иметь вид \(|u|=|v|\), где u и v — алгебраические выражения. Например, \(|x−3|=|2x+1|\).

Как бы мы их решили? Если два алгебраических выражения равны по модулю, то они либо равны друг другу, либо отрицательны друг другу. Свойство для уравнений абсолютного значения говорит, что для любого алгебраического выражения u и положительного действительного числа a , если \(|u|=a\), то \(u=−a\) или \( у=а\).

Это говорит нам, что

\(\ begin{array} {llllll} {\text{if}} &{|u|=|v|} &{} &{} &{} &{} \\ {\text{then]} & {u=−v} &{\text{или}} &{u=v} &{\text{или}} &{u=v} \\ \end{массив}\)

Это приводит нас к следующему свойству уравнений с двумя модулями.

УРАВНЕНИЯ С ДВУМЯ АБСОЛЮТНЫМИ ЗНАЧЕНИЯМИ

Для любых алгебраических выражений, u и v ,

\[\ begin{array} {ll} {\text{if}} &{|u|=|v|} \\ {\text{then}} &{u=-v\text{или}u= v} \\ \номер \конец{массив}\]

Когда мы берем противоположное количество, мы должны быть осторожны со знаками и добавлять круглые скобки там, где это необходимо.

Пример \(\PageIndex{13}\)

Решите: \(|5x−1|=|2x+3|\).

Ответить

\(\begin{массив} {ll} {} &{} &{|5x−1|=|2x+3|} &{} \\ {} &{} &{} &{} \\ {\ text{Напишите эквивалентные уравнения.}} &{5x−1=−(2x+3)} &{\text{or}} &{5x−1=2x+3} \\ {} &{5x−1= −2x−3} &{\text{или}} &{3x−1=3} \\ {\text{Решите каждое уравнение.}} &{7x−1=-3} &{} &{3x=4 } \\ {} &{7x=-2} &{} &{x=43} \\ {} &{x=-27} &{\text{или}} &{x=43} \\ {\ текст {Проверить.}} &{} &{} &{} \\ {\text{Мы оставляем вам чек.}} &{} &{} &{} \\ \end{array}\)

Пример \(\PageIndex{14}\)

Решите: \(|7x−3|=|3x+7|\).

Ответить

\(x=−\frac{2}{5}, \space x=\frac{5}{2}\)

Пример \(\PageIndex{15}\)

Решите: \(|6x−5|=|3x+4|\).

Ответить

\(х=3, х=19\)

Решение абсолютных неравенств – Алгебра среднего уровня

Цели обучения

К концу этого раздела вы сможете:

  • Решение уравнений абсолютного значения
  • Решение абсолютных неравенств с «меньше чем»
  • Решите абсолютные неравенства с «больше чем»
  • Решить приложения с абсолютным значением

Прежде чем начать, пройдите этот тест на готовность.

  1. Оценка:

    Если вы пропустили эту проблему, просмотрите (рисунок).

  2. Введите или для каждой из следующих пар чисел.

    ⓐ ⓑ ⓒ ⓓ

    Если вы пропустили эту проблему, просмотрите (рисунок).

  3. Упростить:

    Если вы пропустили эту проблему, просмотрите (рисунок).

Решение уравнений абсолютного значения

Готовясь к решению уравнений абсолютного значения, мы пересматриваем наше определение абсолютного значения.

Абсолютное значение

Абсолютное значение числа — это его расстояние от нуля на числовой прямой.

Абсолютное значение числа n записывается как и для всех чисел.

Абсолютные значения всегда больше или равны нулю.

Мы узнали, что и число, и его противоположность находятся на одинаковом расстоянии от нуля на числовой прямой. Поскольку они имеют одинаковое расстояние от нуля, они имеют одинаковое абсолютное значение. Например:

находится на расстоянии 5 единиц от 0, поэтому

находится на расстоянии 5 единиц от 0, поэтому

(рисунок) иллюстрирует эту идею.

Числа 5 и оба отстоят от нуля на пять единиц.

Для уравнения мы ищем все числа, которые делают это утверждение верным. Мы ищем числа, расстояние которых от нуля равно 5. Мы только что видели, что и 5, и 5 являются пятью единицами от нуля на числовой прямой. Они являются решениями уравнения.

Решение можно упростить до одного оператора, написав Это читается, « x равно положительному или отрицательному 5».

Мы можем обобщить это до следующего свойства для уравнений абсолютного значения.

Уравнения абсолютного значения

Для любого алгебраического выражения u и любого положительного действительного числа a ,

Помните, что абсолютное значение не может быть отрицательным числом.

Поскольку абсолютное значение всегда положительно, у этого уравнения нет решений.

Оба уравнения говорят нам, что существует только одно решение.

ⓐⓑ нет решения ⓒ 0

ⓐⓑ нет решения ⓒ 0

Чтобы решить уравнение абсолютного значения, мы сначала изолируем выражение абсолютного значения, используя те же процедуры, которые мы использовали для решения линейных уравнений. Как только мы изолируем выражение абсолютного значения, мы перепишем его как два эквивалентных уравнения.

Как решать уравнения абсолютного значения

Решить

Решить:

Решить:

Здесь приведены шаги для решения уравнения абсолютного значения.

Решите уравнения абсолютного значения.

  1. Изолировать выражение абсолютного значения.
  2. Напишите эквивалентные уравнения.
  3. Решите каждое уравнение.
  4. Проверьте каждое решение.

Решить

Решить:

Решить:

Помните, абсолютное значение всегда положительное!

Решить:

Решить:

Решить:

Некоторые из наших уравнений абсолютного значения могут иметь вид, где u и v — алгебраические выражения.Например,

Как бы мы их решили? Если два алгебраических выражения равны по модулю, то они либо равны друг другу, либо отрицательны друг другу. Свойство уравнений абсолютного значения говорит, что для любого алгебраического выражения u и положительного действительного числа a , если тогда или

Это говорит нам, что

Это приводит нас к следующему свойству уравнений с двумя модулями.

Уравнения с двумя абсолютными значениями

Для любых алгебраических выражений, u и v ,

Когда мы берем противоположное количество, мы должны быть осторожны со знаками и добавлять круглые скобки там, где это необходимо.

Решение приложений с абсолютным значением

В производственном процессе часто используются абсолютные неравенства. Товар должен быть изготовлен с почти идеальными характеристиками. Обычно существует определенный допуск отклонения от разрешенных спецификаций. Если расхождение со спецификациями превышает допуск, товар отбраковывается.

Идеальный диаметр стержня, необходимого для машины, составляет 60 мм. Фактический диаметр может отличаться от идеального на миллиметры.Какой диапазон диаметров будет приемлем для заказчика, не вызывая забраковки стержня?

Идеальный диаметр стержня, необходимого для машины, составляет 80 мм. Фактический диаметр может отличаться от идеального на 0,009 мм. Какой диапазон диаметров будет приемлем для заказчика, не вызывая забраковки стержня?

Диаметр стержня может составлять от 79,991 до 80,009 мм.

Идеальный диаметр стержня, необходимого для машины, составляет 75 мм. Фактический диаметр может отличаться от идеального на 0,05 мм. Какой диапазон диаметров будет приемлем для заказчика, не вызывая забраковки стержня?

Диаметр стержня может составлять от 74,95 до 75,05 мм.

Ключевые понятия

  • Абсолютное значение

    Абсолютное значение числа — это его расстояние от 0 на числовой прямой.

    Абсолютное значение числа n записывается как и для всех чисел.

    Абсолютные значения всегда больше или равны нулю.

  • Уравнения абсолютного значения

    Для любого алгебраического выражения u и любого положительного действительного числа a ,

    Помните, что абсолютное значение не может быть отрицательным числом.

  • Как решать уравнения абсолютного значения
    1. Изолировать выражение абсолютного значения.
    2. Напишите эквивалентные уравнения.
    3. Решите каждое уравнение.
    4. Проверьте каждое решение.
  • Уравнения с двумя абсолютными значениями

    Для любых алгебраических выражений, u и v ,

  • Неравенства абсолютного значения с или

    Для любого алгебраического выражения u и любого положительного действительного числа a ,

  • Как решать неравенства абсолютного значения с помощью или
    1. Изолировать выражение абсолютного значения.
    2. Запишите эквивалентное составное неравенство.

    3. Решите составное неравенство.
    4. График решения
    5. Запишите решение, используя интервальную нотацию
  • Неравенства абсолютного значения с или

    Для любого алгебраического выражения u и любого положительного действительного числа a ,

  • Как решать неравенства абсолютного значения с помощью или
    1. Изолировать выражение абсолютного значения.
    2. Запишите эквивалентное составное неравенство.

    3. Решите составное неравенство.
    4. График решения
    5. Запишите решение, используя интервальную нотацию

Раздел Упражнения

Практика делает совершенным

Решение уравнений абсолютного значения

В следующих упражнениях решите.

ⓐⓑ нет решения ⓒ

ⓐⓑ нет решения ⓒ

Решение абсолютных неравенств с «меньше чем»

В следующих упражнениях решите каждое неравенство. Нарисуйте график решения и запишите решение в интервальной записи.

Решение абсолютных неравенств с «больше чем»

В следующих упражнениях решите каждое неравенство. Нарисуйте график решения и запишите решение в интервальной записи.

В следующих упражнениях решите. Для каждого неравенства также постройте график решения и запишите решение в интервальной записи.

Решение приложений с абсолютным значением

В следующих упражнениях решите.

Птицеферма в идеале производит 200 000 яиц в день. Но это общее количество может варьироваться на целых 25 000 яиц. Какова максимальная и минимальная ожидаемая производительность фермы?

Производитель органических соков в идеале производит 215 000 бутылок в день. Но эта сумма может варьироваться на целых 7500 бутылок. Какова максимальная и минимальная ожидаемая производительность компании по розливу?

Ожидаемый объем производства от минимального до максимального составляет от 207 500 до 2 225 000 бутылок

Чтобы обеспечить соблюдение закона, Мигель регулярно превышает вес своих лепешек на 0.5 грамм. Он только что получил отчет, в котором говорилось, что он может терять до 100 000 фунтов стерлингов в год, используя эту практику. Теперь он планирует закупить новое оборудование, гарантирующее толщину лепешки в пределах 0,005 дюйма. Если идеальная толщина лепешки составляет 0,04 дюйма, какая толщина лепешки будет гарантирована?

В пекарне Lilly идеальный вес буханки хлеба составляет 24 унции. По закону реальный вес может отличаться от идеального на 1,5 унции. Какой диапазон веса будет приемлем для инспектора без штрафа пекарни?

Допустимый вес 22.от 5 до 25,5 унций.

Письменные упражнения

Напишите графическое описание абсолютного значения числа.

Своими словами объясните, как решить абсолютное неравенство,

Самопроверка

ⓐ После выполнения упражнений используйте этот контрольный список, чтобы оценить свое мастерство выполнения целей этого раздела.

ⓑ Что этот контрольный список говорит вам о вашем мастерстве в этом разделе? Какие шаги вы предпримете для улучшения?

Упражнения по обзору главы

Использование общей стратегии для решения линейных уравнений

Решение уравнений с использованием общей стратегии решения линейных уравнений

В следующих упражнениях определите, является ли каждое число решением уравнения.

В следующих упражнениях решите каждое линейное уравнение.

Классифицировать уравнения

В следующих упражнениях классифицируйте каждое уравнение как условное уравнение, тождество или противоречие, а затем сформулируйте решение.

противоречие; нет решения

Решение уравнений с дробями или десятичными коэффициентами

В следующих упражнениях решите каждое уравнение.

Используйте стратегию решения проблем

Использование стратегии решения проблем Word

В следующих упражнениях решите, используя стратегию решения текстовых задач.

Три четверти присутствующих на концерте – дети. Если детей 87, сколько человек на концерте?

В группе девять саксофонистов. Количество играющих на саксофоне в два раза меньше, чем у играющих на тубе. Найдите количество играющих на тубе.

Решить числовые задачи со словами

В следующих упражнениях решите задачу с числовыми словами.

Сумма числа и трех равна сорок одному.Найдите число.

Одно число на девять меньше другого. Их сумма равна минус двадцати семи. Найдите числа.

Одно число в два раза больше другого, чем в четыре раза. Их сумма равна минус тринадцать. Найдите числа.

Сумма двух последовательных целых чисел равна Найдите числа.

Найдите три последовательных четных целых числа, сумма которых равна 234.

Найдите три последовательных нечетных целых числа, сумма которых равна 51.

На сберегательном счете Коджи 5502 фунта стерлингов. Это на 30 фунтов меньше, чем в шесть раз больше суммы на его текущем счете. Сколько денег у Коджи на текущем счету?

Решение процентных приложений

В следующих упражнениях переведите и решите.

Какое число составляет 67% от 250?

12,5% от какого числа будет 20?

Сколько процентов от 125 составляет 150?

В следующих упражнениях решите.

Счет за обед Дино составил 19,45 фунтов стерлингов. Он хотел оставить 20% от суммы счета в качестве чаевых.Сколько должны быть чаевые?

Долорес купила на распродаже детскую кроватку за 350 фунтов стерлингов. Цена продажи составила 40% от первоначальной цены. Какова была первоначальная цена кроватки?

Джейден зарабатывает 2680 фунтов стерлингов в месяц. Он платит 938 фунтов стерлингов в месяц за аренду. Какой процент от его месячной зарплаты уходит на аренду?

Годовой оклад Ангела увеличился с 55 400 до 56 785 фунтов стерлингов. Найдите процентное изменение.

Ежемесячный счет Ровены за бензин снизился с 83,75 фунтов стерлингов в прошлом месяце до 56 фунтов стерлингов.95 в этом месяце. Найдите процентное изменение.

Эммет купил пару обуви на распродаже со скидкой 40% по сравнению с первоначальной ценой 138 фунтов стерлингов. Найдите ⓐ сумму скидки и ⓑ цену продажи.

Лейси купила пару ботинок на распродаже за 95 фунтов стерлингов. Первоначальная цена ботинок составляла 200 фунтов стерлингов. Найдите ⓐ сумму скидки и ⓑ ставку скидки. (При необходимости округлить до десятых долей процента.)

ⓐ ?105 ⓑ

Нга и Лорен купили сундук на барахолке за 50 фунтов стерлингов.Они переделали его, а затем добавили наценку 350%. Найдите ⓐ сумму наценки и ⓑ прейскурантную цену.

Решение простых процентов

В следующих упражнениях решите.

Уинстон положил 3294 фунта на банковский счет с процентной ставкой 2,6% Сколько процентов было получено за пять лет?

Мойра заняла у дедушки 4500 фунтов, чтобы заплатить за первый год обучения в колледже. Три года спустя она выплатила 4500 фунтов плюс 243 процента.Какова была процентная ставка?

В заявлении Хайме о кредите на холодильник говорилось, что он заплатит 1026 фунтов стерлингов в виде процентов за четырехлетний кредит под 13,5%. Сколько Хайме занял на покупку холодильника?

Решение формулы для конкретной переменной

Решение формулы для определенной переменной

В следующих упражнениях решите формулу для указанной переменной.

Решить формулу

для л .

Решить формулу

для т .

Решить формулу

за и .

Использование формул для решения геометрических задач

В следующих упражнениях решите с помощью геометрической формулы.

Какова высота треугольника с площадью квадратных метров и основанием 9 метров?

Мера наименьшего угла в прямоугольном треугольнике меньше, чем мера следующего большего угла. Найдите меры всех трех углов.

Периметр треугольника равен 97 футам.Одна сторона треугольника на одиннадцать футов больше наименьшей стороны. Третья сторона на шесть футов больше, чем в два раза меньше наименьшей стороны. Найдите длины всех сторон.

Найдите длину гипотенузы.

Найдите длину недостающей стороны. Округлите до десятых, если необходимо.

Серджио нужно прикрепить провод для крепления антенны к крыше своего дома, как показано на рисунке. Антенна имеет высоту восемь футов, а у Серхио 10 футов провода.На каком расстоянии от основания антенны можно прикрепить провод? Приблизительно с точностью до десятых, если необходимо.

Сон строит стеллажи в своем гараже. Полки имеют ширину 36 дюймов и высоту 15 дюймов. Он хочет установить диагональную распорку сзади, чтобы стабилизировать полки, как показано на рисунке. Какой длины должен быть бандаж?

Длина прямоугольника на 12 см больше ширины. Периметр 74см. Найдите длину и ширину.

см,

см

Ширина прямоугольника в три раза больше его длины.Периметр 96 дюймов. Найдите длину и ширину.

Периметр треугольника равен 35 футам. Одна сторона треугольника на пять футов длиннее второй. Третья сторона на три фута длиннее второй. Найдите длину каждой стороны.

Решение задач смешанного и равномерного движения

Решить задачи с монетами

В следующих упражнениях решите.

Полетт имеет 140 фунтов стерлингов банкнотами номиналом 5 и 10 фунтов стерлингов. Количество банкнот номиналом 10 фунтов стерлингов на единицу меньше, чем удвоенное количество банкнот номиналом 5 фунтов стерлингов.Сколько у нее каждого?

У Ленни есть 3,69 фунта стерлингов в пенни, десятицентовике и четвертаке. Количество копеек на три больше, чем количество десятицентовиков. Количество четвертаков в два раза больше количества десятицентовиков. Сколько у него каждой монеты?

девять пенни, шесть десятицентовиков, 12 четвертаков

Решение проблем с билетами и штампами

В следующих упражнениях решите задачу с каждым билетом или штампом.

Билеты на баскетбольный матч стоят 2 евро для студентов и 5 евро для взрослых.Количество студентов было на троих меньше, чем в 10 раз больше, чем взрослых. Общая сумма денег от продажи билетов составила 619 фунтов стерлингов. Сколько билетов каждого было продано?

На концерт джаз-бэнда было продано 125 билетов на общую сумму 1022 евро. Студенческие билеты стоят 6 евро каждый, а обычные входные билеты стоят 10 евро каждый. Сколько билетов каждого вида было продано?

Юми потратила 34,15 евро на покупку марок. Количество купленных ею марок по 0,56 фунта стерлингов было на 10 меньше, чем в четыре раза больше, чем количество марок по 0,56 фунта стерлингов.41 марка. Сколько штук каждого она купила?

Решение задач на смешанные слова

В следующих упражнениях решите.

Marquese готовит 10 фунтов смеси из изюма и орехов. Изюм стоит 3,45 фунтов стерлингов за фунт, а орехи — 7,95 фунтов стерлингов за фунт. Сколько фунтов изюма и сколько фунтов орехов должен использовать Маркиз для смеси для троп, чтобы он стоил ему 6,96 фунтов стерлингов за фунт?

фунтов изюма,

фунтов орехов

Эмбер хочет положить плитку на фартук своей кухни.Ей понадобится 36 квадратных футов плитки. Она будет использовать базовые плитки по цене 8 фунтов стерлингов за квадратный фут и плитки декораторов по цене 20 фунтов стерлингов за квадратный фут. Сколько квадратных футов каждой плитки она должна использовать, чтобы общая стоимость фартука составила 10 фунтов стерлингов за квадратный фут?

Энрике занял 23 500 фунтов стерлингов, чтобы купить машину. Он платит своему дяде 2% процентов на 4500 фунтов стерлингов, которые он у него одолжил, и платит банку 11,5% процентов на оставшуюся часть. Какую среднюю процентную ставку он платит на общую сумму 23 500 фунтов стерлингов? (Округлите ответ до десятых долей процента. )

 *** QuickLaTeX не может скомпилировать формулу:
9.7\текст{%}

*** Сообщение об ошибке:
Файл завершился при сканировании с использованием \text@.
Экстренная остановка.

 

Решение задач равномерного движения

В следующих упражнениях решите.

Когда Гейб едет из Сакраменто в Реддинг, ему требуется 2,2 часа. Эльзе требуется два часа, чтобы проехать такое же расстояние. Скорость Эльзы на семь миль в час больше, чем скорость Гейба. Найдите скорость Гейба и скорость Эльзы.

Луэллен и Трейси встретились в ресторане на дороге между Чикаго и Нэшвиллом. Луэллен выехал из Чикаго и ехал 3,2 часа в сторону Нэшвилла. Трейси выехала из Нэшвилла и 4 часа ехала в сторону Чикаго со скоростью на одну милю в час больше, чем скорость Луэллена. Расстояние между Чикаго и Нэшвиллом составляет 472 мили. Найдите скорость Луэллен и скорость Трейси.

Луэллен 65 миль в час, Трейси 66 миль в час

Два автобуса отправляются из Амарилло одновременно. Автобус из Альбукерке едет на запад по I-40 со скоростью 72 мили в час, а автобус из Оклахома-Сити едет на восток по I-40 со скоростью 78 миль в час.Через сколько часов расстояние между ними составит 375 миль?

Кайл греб на лодке против течения 50 минут. Ему потребовалось 30 минут, чтобы грести обратно вниз по течению. Его скорость вверх по течению на две мили в час меньше, чем его скорость вниз по течению. Найдите скорости Кайла по течению и по течению.

вверх по течению 3 мили в час, вниз по течению 5 миль в час

В 6:30 Девон вышла из дома и до 7:30 ехала на велосипеде по ровной дороге. Потом она поехала в гору и ехала до 8:00. Всего она проехала 15 км.Ее скорость на ровной дороге была на три мили в час больше, чем скорость на подъеме. Найдите скорость Девона на ровной дороге и в гору.

Энтони проехал из Нью-Йорка в Балтимор, что составляет 192 мили. Он уехал в 3:45, и до 5:30 пробки были в пробках. До конца пути движение было легким, и он прибыл в 7:30. Его скорость в легком потоке была на четыре мили в час больше, чем в два раза больше, чем в плотном потоке. Найдите скорость движения Энтони в плотном и слабом трафике.

интенсивное движение 32 мили в час, легкое движение 66 миль в час

Решение линейных неравенств

Графические неравенства на числовой прямой

В следующих упражнениях нарисуйте неравенство на числовой прямой и запишите в виде интервалов.

Решение линейных неравенств

В следующих упражнениях решите каждое неравенство, нарисуйте решение на числовой прямой и запишите решение в виде интервалов.

Перевести слова в неравенство и решить

В следующих упражнениях переведите и решите. Затем запишите решение в интервальной записи и начертите на числовой прямой.

На пять больше, чем z не более 19.

На три меньше c не менее 360.

Отрицательное два раза a не более восьми.

Решение приложений с линейными неравенствами

В следующих упражнениях решите.

Еженедельный бюджет Джулианны на еду для ее семьи составляет 231 фунт стерлингов. Если она планирует выделять одинаковую сумму на каждый из семи дней недели, какова максимальная сумма, которую она может тратить на еду каждый день?

Рохелио рисует акварелью. Он получил подарочную карту на 100 фунтов стерлингов в магазин товаров для творчества и хочет использовать ее для покупки холстов размером 12″ × 16″. Каждый холст стоит 10,99 фунтов стерлингов.Какое максимальное количество полотен он может купить на свою подарочную карту?

Бриане предложили работу продавца в другом городе. Предложение было за 42 500 фунтов стерлингов плюс 8% от ее общего объема продаж. Чтобы переезд стоил того, Бриана должна иметь годовой оклад не менее 66 500 фунтов стерлингов. Каким должен быть ее общий объем продаж, чтобы она переехала?

Автомобиль Рене обходится ей в 195 фунтов стерлингов в месяц плюс 0,09 фунтов стерлингов за милю. Сколько миль может проехать Рене, чтобы ее ежемесячные расходы на машину не превышали 250 фунтов стерлингов?

Коста — бухгалтер.Во время налогового сезона он берет 125 фунтов за простую налоговую декларацию. Его расходы на покупку программного обеспечения, аренду офиса и рекламу составляют 6000 фунтов стерлингов. Сколько налоговых деклараций он должен подать, если он хочет получить прибыль не менее 8000 фунтов стерлингов?

Дженна планирует пятидневный отпуск на курорте с тремя своими друзьями. Это обойдется ей в 279 фунтов стерлингов за перелет, 300 фунтов стерлингов за еду и развлечения и 65 фунтов стерлингов в день за ее долю в отеле. Она накопила 550 фунтов стерлингов на отпуск и может зарабатывать 25 фунтов стерлингов в час, работая ассистентом в фотостудии своего дяди.Сколько часов она должна работать, чтобы ей хватило денег на отпуск?

Решение сложных неравенств

Решение сложных неравенств с помощью «и»

В каждом из следующих упражнений решите каждое неравенство, нарисуйте решение и запишите решение в виде интервалов.

и

и

Решение сложных неравенств с помощью «или»

В следующих упражнениях решите каждое неравенство, нарисуйте решение на числовой прямой и запишите решение в виде интервалов.

или

или

Решение приложений с составными неравенствами

В следующих упражнениях решите.

Лиам играет в числовую игру со своей сестрой Одри. Лиам думает о числе и хочет, чтобы Одри угадала его. В пять раз больше, чем в три раза, ее число находится между 2 и 32. Напишите составное неравенство, которое показывает диапазон чисел, о которых может думать Лиам.

Элуиза создает прямоугольный сад на заднем дворе.Длина сада 12 футов. Периметр сада должен быть не менее 36 футов и не более 48 футов. Используйте составное неравенство, чтобы найти диапазон значений ширины сада.

Решение абсолютных неравенств

Решение уравнений абсолютного значения

В следующих упражнениях решите.

Решение абсолютных неравенств с «меньше чем»

В следующих упражнениях решите каждое неравенство.Нарисуйте график решения и запишите решение в интервальной записи.

Решение абсолютных неравенств с «больше чем»

В следующих упражнениях решите. Нарисуйте график решения и запишите решение в интервальной записи.

Решение приложений с абсолютным значением

В следующих упражнениях решите.

Пивовару крафтового пива требуется 215 000 бутылок в день. Но эта сумма может варьироваться на целых 5000 бутылок. Каково максимальное и минимальное ожидаемое использование в компании по розливу?

Минимальное и максимальное ожидаемое использование составляет от 210 000 до 220 000 бутылок

В Fancy Grocery идеальный вес буханки хлеба составляет 16 унций. По закону реальный вес может отличаться от идеального на 1,5 унции. Какой диапазон веса будет приемлем для инспектора без штрафа пекарни?

Практический тест

В следующих упражнениях решите каждое уравнение.

противоречие; нет решения

Решить формулу

за и .

В следующих упражнениях нарисуйте неравенство на числовой прямой и запишите в виде интервалов.

В следующих упражнениях решите каждое неравенство, нарисуйте решение на числовой прямой и запишите решение в виде интервалов.

и

или

В следующих упражнениях переведите в уравнение или неравенство и решите.

На четыре меньше, чем в два раза x равно 16.

Найдите длину недостающей стороны.

Одно число в четыре раза больше другого. Их сумма равна Найдите числа.

Сумма двух последовательных нечетных целых чисел равна Найдите числа.

Маркус купил телевизор на распродаже за 626,50 фунтов стерлингов. Первоначальная цена телевизора составляла 895 фунтов стерлингов. Найдите ⓐ сумму скидки и ⓑ ставку скидки.

У Бониты в кармане 2,95 фунта десятицентовика и четвертака. Если у нее на пять десятицентовиков больше, чем четвертаков, сколько у нее каждой монеты?

Ким готовит восемь галлонов пунша из фруктового сока и содовой. Фруктовый сок стоит 6,04 фунта стерлингов за галлон, а газировка стоит 4,28 фунта стерлингов за галлон. Сколько фруктового сока и сколько газировки она должна использовать, чтобы пунш стоил 5 фунтов стерлингов.71 за галлон?

Размер одного угла треугольника в два раза больше наименьшего угла. Мера третьего угла в три раза больше меры наименьшего угла. Найдите меры всех трех углов.

Длина прямоугольника на пять футов больше ширины, чем в четыре раза. Периметр 60 футов. Найдите размеры прямоугольника.

Два самолета одновременно вылетели из Далласа. Один направляется на восток со скоростью 428 миль в час.Другой самолет летит на запад со скоростью 382 мили в час. Через сколько часов расстояние между ними составит 2025 миль?

часа

Леон проехал от своего дома в Цинциннати до дома своей сестры в Кливленде, расстояние 252 мили. Это заняло у него несколько часов. В течение первых получаса у него было плотное движение, а в остальное время его скорость была на пять миль в час меньше, чем вдвое его скорость в плотном потоке. Какова была его скорость в плотном потоке?

Бюджет Сары составляет 1000 фунтов стерлингов на костюмы для 18 членов ее труппы музыкального театра.Сколько максимум она может потратить на каждый костюм?

Максимум 55,56 фунтов стерлингов за костюм.

Калькулятор абсолютного значения — Найдите абсолютные значения для неравенств

Онлайн-калькулятор абсолютных значений предоставляет абсолютные значения реальных значений и функций отдельно для заданных значений. Этот калькулятор абсолютных неравенств предоставляет всю важную информацию о неравенствах и абсолютных функциях. Здесь вы можете узнать больше о том, как находить неравенства абсолютного значения и многое другое.

Что такое абсолютные значения?

В математике абсолютная величина или модуль | х | действительного числа х является неотрицательным значением х, независимо от его знака. То есть | х | = x означает положительный x, | х | = -x означает отрицательный x (в данном случае -x положительный), а | 0 | =0. Другими словами, уравнение абсолютного значения определяется как уравнение, содержащее выражение абсолютного значения. Например, |х + 8| = 12. Здесь | х + 8 | является абсолютным выражением. Калькулятор абсолютного значения указывает только разницу между указанным числом и нулем, потому что отрицательные значения здесь не допускаются.

График абсолютных значений:

Общая форма уравнения записи абсолютного значения:

F(x) = k + a |x – h|

Это уравнение, используемое калькулятором графика абсолютных значений, где k и h говорят о том, как график смещается по вертикали и горизонтали. Переменная «a» говорит нам, как далеко простирается график значений по вертикали, и открывается ли график вниз или вверх.

Однако онлайн-калькулятор среднего абсолютного отклонения поможет вам найти абсолютное отклонение заданного числа от среднего, медианы или любого другого числа.

Уравнения абсолютного значения и неравенства абсолютного значения:

Обычно это значение находится путем решения уравнения абсолютного значения, а уравнения с этими значениями называются абсолютными уравнениями; если вместо знака равенства (=) у нас есть знак меньше (<), меньше или равно (≤), больше (<) или больше или равно (≥), то это будет называться абсолютным ценностное неравенство.

В обоих случаях калькулятор абсолютных неравенств работает одинаково: при решении уравнения максимально упрощает уравнение или числа.Когда нам приходится иметь дело с этими значениями, мы изолируем его сбоку от знака и делим на возможные варианты: положительные и отрицательные. Этот процесс одинаков для абсолютных неравенств и уравнений абсолютных значений.

Как решать уравнения абсолютного значения?

Решать уравнения абсолютного значения так же просто, как использовать обычные линейные уравнения. Единственным дополнительным важным шагом является разделение исходного уравнения абсолютного значения на две части: положительную и отрицательную (±) составляющие.

  • Символ ∣x∣ должен быть положительным. Например, |x∣→+ ∣x∣.
  • x в символе абсолютного значения | | может быть любое уравнение.
  • Для решения правая часть уравнения должна быть положительным числом или нулем.
  • Если «а» справа отрицательное, то результатов нет.

Пример:

Решить уравнение абсолютного значения − x = −9.

Решение:

Не спешите делать вывод, что это уравнение не имеет решения. Хотя правая часть уравнения является отрицательным числом, само выражение абсолютного значения должно быть положительным числом. это неправильно?

  • | х | знак должен быть положительным. Подчеркнуть | х ∣ → + ∣ х∣.
  • Калькулятор абсолютного значения удаляет знак минус из знака абсолютного значения, прежде чем мы сможем продолжить.
  • Коэффициент этого уравнения равен -1. Таким образом, калькулятор уравнения абсолютного значения делит обе части уравнения на это значение, чтобы удалить отрицательный знак.

-|х| = -9

-1 . |х| = -9

-1 . |х| / -1 = -9/-1

|х| = 9

  • Поскольку выражение абсолютного значения и число положительное, калькулятор уравнений абсолютного значения использует процесс, чтобы разделить их на два уравнения.

|х| = а => х = +а, х = -а

  • Итак, решение задачи становится

|х| = 9 => х = 9, х = -9

Однако онлайн-калькулятор гиперболы поможет вам определить центр, эксцентриситет, фокальный параметр, мажор и асимптоту для заданных значений в уравнении гиперболы.

Как решать абсолютные неравенства?

Решение абсолютных неравенств шаг за шагом с последующими шагами:

  • Сначала в левой части неравенства выделите выражение абсолютного значения.
  • Если число по другую сторону знака неравенства отрицательное, то уравнение не имеет решения. Используйте знак каждой стороны вашего неравенства, чтобы решить, какой из этих случаев имеет место. Если в другой части неравенства число положительное, то переходим к следующему шагу.
  • Удалите абсолютные значения, установив составное неравенство. Тип знака неравенства в задаче описывает, как установить неравенство.

Если в заданной задаче стоит знак больше, то задайте составное неравенство как:

(абсолютное значение) < (число на другой стороне)

или

(Абсолютное значение) > (число на другой стороне)

Выполните те же действия для знака ≥.

Если ваше абсолютное значение меньше числа, то установите составное неравенство из трех частей, которое выглядит следующим образом:

(номер на другой стороне) < (внутри абсолютного значения количества) < (номер на другой стороне)

Та же установка используется для знака ≤

  • Решите неравенства с помощью калькулятора абсолютного неравенства.

Как работает калькулятор абсолютного значения?

Онлайн-калькулятор уравнения абсолютного значения изменяет входные значения на положительное число и проверяет определенные точки графика абсолютного значения и уравнения, выполнив следующие шаги:

Ввод:
  • Нажмите на вкладку «Число», если вы хотите выполнить математическую операцию над заданным числом, или выберите вкладку «для уравнения», если вы хотите решить уравнение.
  • Теперь подставьте указанные значения в поля выбранной вкладки.
  • Нажмите кнопку расчета для решения.

Выход:
  • Калькулятор абсолютного значения решает число или уравнение для абсолютных значений в соответствии с вашим выбором.

Часто задаваемые вопросы:

Что такое правило абсолютного значения?

Абсолютное значение числа — это расстояние от нуля на числовой прямой.

Почему абсолютное значение никогда не бывает отрицательным?

Абсолютное значение представляет собой расстояние, но не содержит информации о направлении. Поскольку направление игнорируется, абсолютное значение любого числа может быть только положительным или равным нулю, но не отрицательным.

Вывод:

Используйте этот онлайн-калькулятор абсолютного значения для решения уравнений или чисел, чтобы лучше понять всю концепцию. В алгебре не так-то просто иметь дело с абсолютными значениями. Многие математики считают, что понятие абсолютных величин несложно, но требует глубокого понимания и большего интереса.

Артикул:

Из источника Википедии: Терминология и обозначения, Определение и свойства, Комплексные числа, Доказательство комплексного неравенства треугольника, Функция абсолютного значения.
Из источника HMH: что означает абсолютное значение, примеры и уравнения абсолютного значения, материалы, стандарты, необходимые навыки и концепции.

Из источника Purple Math: Связь с функцией знака, Производная, Первообразная, Расстояние, Упорядоченные кольца, Поля, Векторные пространства.

Как решать абсолютные неравенства | Алгебра

Как решать абсолютные неравенства

Шаг 1 : Чтобы решить неравенство абсолютного значения алгебраически, сначала перепишите неравенство без символов абсолютного значения.

{eq}|ax+b|

{eq}|ax+b|\le c \Rightarrow -c\le ax+b\le c {/экв}

{eq}|ax+b|>c \Rightarrow ax+b<-c\text{ или }ax+b>c {/экв}

{eq}|ax+b|\ge c \Rightarrow ax+b\le -c\text{ или }ax+b\ge c {/экв}

Шаг 2 : Решите полученные линейные неравенства путем выделения переменной.

Шаг 3 : Для графического решения абсолютного неравенства установите

{eq}y=\text{левая часть} {/экв}

и

{eq}y=\text{правая часть} {/экв}

и с помощью графического калькулятора начертите два уравнения.

Шаг 4 : Используйте графики из Шага 3, чтобы определить, для каких значений {eq}x {/eq} первый график выше (в случае {eq}> {/eq} или {eq}\ge {/eq}) или ниже (в случае {eq}< {/eq} или {eq}\le {/eq}) второй график. При необходимости включите конечные точки.

Шаг 5 : Сравните решения каждого метода и убедитесь, что они одинаковы.

Как решать абсолютные неравенства: словарный запас

Абсолютное значение : Абсолютное значение A, обозначаемое {eq}|A| {/eq} — расстояние между {eq}A {/экв} и {экв}0 {/eq} в числовой строке.

Неравенство абсолютного значения : If {eq}|A|

Если {экв}|A|>d {/eq}, то расстояние между {eq}A {/экв} и {экв}0 {/eq} больше, чем d. Следовательно, A либо больше d, либо меньше -d: {eq}A<-d \text{ или }A>d {/экв}.То же самое верно для {eq}|A|\ge d {/eq}, за исключением того, что включены конечные точки: {eq}|A|\le -d\text{ или }|A|\ge d {/экв}.

Интервальная запись : Интервальная запись используется для выражения решений неравенств. Это четыре возможных набора решений, с которыми мы можем столкнуться при работе с абсолютными неравенствами типа, описанного в этом уроке:

{eq}a\le x\le b\Leftrightarrow [a,b] \\ a b \Leftrightarrow (-\infty,a)\cup (b,\infty) {/экв}

Базовый график абсолютного значения :

Теперь давайте потренируемся решать два абсолютных неравенства, одно с ограниченным решением, а другое с неограниченным решением.

Как решать абсолютные неравенства: пример с ограниченным решением

Решите для {eq}x {/eq} как алгебраически, так и графически:

{экв}|2x-5|\le 7. {/экв}

Шаг 1 : Чтобы решить алгебраически, мы перепишем абсолютное неравенство как

{экв}-7\le 2x-5\le 7. {/экв}

Шаг 2 : Приведенное выше составное неравенство эквивалентно

{eq}-7\le 2x-5\text{ и}2x-5\le 7. {/экв}

Для {экв}x Чтобы {/eq} было решением, оно должно удовлетворять обоим неравенствам.Мы можем решить оба неравенства одновременно.

Чтобы изолировать {eq}x {/eq} в середине сложного неравенства сначала добавьте {eq}5 {/eq} ко всем трем частям составного неравенства:

{экв}-2\le 2x\le 12. {/экв}

Затем разделите все три части составного неравенства на {eq}2 {/eq}, чтобы получить окончательный результат:

{экв}-1\le x\le 6. {/экв}

Мы можем выразить набор решений, используя интервальную запись, как

{экв}[-1,6]. {/экв}

Мы используем квадратные скобки для включения {eq}-1 {/экв} и {экв}6 {/eq}, потому что символ неравенства, используемый в наборе решений, включает в себя равенство.

Шаг 3 : Для графического решения абсолютного неравенства мы сначала устанавливаем {eq}y {/eq} равно каждой части неравенства

{экв}y=|2x-5| \\ у=7. {/экв}

Шаг 4 : Далее мы наносим на график эти два уравнения:

Из графиков видно, что {eq}y=|2x-5| {/eq} ниже (меньше или равно) {eq}y=7 {/экв} для {экв}х {/eq} от -1 до 6:

{eq}-1\le x\le 6\text{ или }[-1,6].{/экв}

Опять же, мы включаем конечные точки, потому что символ неравенства включает в себя равенство. Мы видим, что набор решений, полученный алгебраически, совпадает с набором решений, полученным графически, что и требовалось.

Как решать абсолютные неравенства: пример с неограниченным решением

Решите для {eq}x {/eq} как алгебраически, так и графически:

{экв}|x+4|> 2. {/экв}

Шаг 1 : Переписывая неравенство без символов модуля, получаем

{экв}x+4<-2\текст{ или }x+4>2.{/экв}

Шаг 2 : Опять же, мы можем решить эти два неравенства одновременно, но мы должны оставить между ними слово «или». Другими словами, {экв}x {/eq} является решением, если оно удовлетворяет либо первому, либо второму неравенству.

Добавить комментарий

Ваш адрес email не будет опубликован. Обязательные поля помечены *